Ask Anna

Your questions answered here...

STAY CURIOUS!!!

This page is devoted to the daily questions our ladies have asked....See responses and links below.

If Anna doesn't know the answer... she will ask someone who knows!

Questions from 2021 Summer Camp

Friday June 18


Q: What does NM Tech have in terms of dual credit/underage admissions programs?

A: New Mexico Tech offers dual credit to students who attend high schools in the general area of the campus. Unfortunately we are not able to offer dual to everyone, but you are still always welcome to take summer courses with us. New Mexico Tech does also have a yearly summer program, The Summer STEM Experience, for rising juniors and seniors. We are looking to hopefully expand that program and open it up to all high school students in the coming years. You can always feel free to reach out to the Office of Admission for more information about that program. J. Cervantes

Q: What impact does volunteer hours have at UNM?

A: Volunteering can be very beneficial once you start to look for internship or job opportunities. Volunteering also enhances your academic career and helps you network with potential employers. Additionally, programs like the BA/MD or Nursing highly encourage volunteering for their application process. P. Nunez

Q: What impact does volunteer hours have at NMTech?

A: For admission, volunteer hours do not have any impact. We try to keep our admission process very straight forward which is why it is strictly based on GPA and test scores. Volunteer hours can help you out for other things though, such as non-institutional scholarships, future internships, on and off-campus jobs, etc. So do not feel like they are at all useless! J. Cervantes

Q: What impact does volunteer hours have at NMSU?

Q: On the material return form, what do you mean by "yes" and "no"?

Q: Athena - Where would the pocket nuclear reactors go to? And how long do these nuclear reactors last?

A: I am certain that they will be trained. However, similar to how spent nuclear fuel is transported, these microreactotors will be very safe and and their transportation will have to adhere to strict regulations to ensure everyone's safety.

Q: Priscilla Nunez- Can you provide me with more information about the BA/MD program? I interested in pre-med so I would love to learn about the BA/MD program.

A: The BA/MD program allows high school seniors to apply to our Medical School. If a student is accepted into the BA/MD program they will have a reserve seat into our Medical School, will obtain a full ride scholarship for the undergraduate program, and will get mentorship. You can learn more about the program by visiting https://hsc.unm.edu/medicine/education/ba-md/. P. Nunez

Q: How old do we have to be for us to apply for an internship?

A: Most internships do not have an age criteria but might have a classification (sophomore, junior, etc.) requirement. I recommend looking for internship opportunities once you start at UNM. There are a lot of opportunities provided by our Career Services Office, https://career.unm.edu/.P. Nunez

A: At New Mexico Tech many of our students start internships the summer between their freshman and sophomore years. Because of this they are able to gain a great amount of experience during the entire course of their undergraduate degree! Many of these students are continuously asked back by the companies that they intern with every year as well, and many are even offered jobs by these companies at the end of their final summer before they graduate from New Mexico Tech. In the past I have even seen internship opportunities for high school students though, so don't be afraid to look around. You never know what kind of amazing opportunities that you can find! J. Cervantes


Thursday June 17

Q: when going into the military at the age you did, is it possible to do a different job that doesn't include the fighting?"

A: I had training to help me survive any situation (I was in the Marines after all), this said on my day to day job there was no fighting as I was an electronics tech. There are many other jobs one can do mostly using your brain and I can elaborate further (ie translator etc) . Also, the military does not just go to war but there are branches that do live saving missions; help citizens after a big flood/fire, medical emergencies, earthquake search and rescue etc etc Astrid Morreale

Q: Did being in the military benefit your educational journey? Besides financially.

A: Yes, of course! there is very little in this world that I think I cannot do, I also have a larger view on things than my peers . I am known for getting things done and efficient and that is probably thanks to my military training. I also traveled to Japan and learned Japanese and visited many many states within the USA. Today I still have many friends that I keep in contact from the time I was in the military.-Astrid Morreale

Q: Did you do anything STEM related in the Marines?

A: Lots of STEM outreach, specially in racially segregated areas in the USA and with kids with disabilities in Japan. -Astrid Morreale

Q: How was you're experience with the military? I was considering taking the same route as you, but I have some doubts that maybe you could clear up.

A: Well it depends what your ultimate goal is. I wanted to finance my education, gain financial independence while getting a different view of the world along the way.- Astrid Morreale

Q: (Stacey Copp) Why doesn't pen ink dissolve like highlighters or color markers?

A: Great question! Anna did an experiment to best illustrate the answer. Here is a picture to show the answer. Basically, permanent markers and pens often have inks which don't dissolve in water (are not soluble in water), but certain organic solvents can dissolve the ink to enable chromatography. So if you need to clean sharpie off of your skin or clothes, maybe try hand sanitizer!

Q: Are there any national societies for asian-american students?

A; yes the most famous one is SASE:

https://www.saseconnect.org/

Astrid Morreale

Q: Is there a paper that gives us information on what we get to decide what we get to keep or have at the end of our camp? Cause i need more info.

A: Look into the Materials Return google form sent on 6/17. If you follow the instructions, you will see what you can keep and what you need to return


Q: I would like to ask Alex Miera if she is worried that the research she does, especially any research going to the military/ the iron man, will go into unethical projects in the future.


Q: Why does the Sandia Hand only have 4 fingers?

A: The pinky is actually unnecessary when picking up and gripping things. Usually when you go to pick something up or move it, you naturally try to get your pinky out of the way when doing these tasks. That’s why the Sandia hand only has 3 fingers and a thumb. C. Gilbert


Q: How old do we have to be to apply for internships? also is there any more programs like this that you know of?

A: To apply for the High School Internship Program at Los Alamos National Laboratory, you must at least 16 years old at the time the internship begins AND have senior standing in high school. The earliest a student can apply is the second semester of their junior year in high school, in order to intern the summer after completing their junior year, when they have senior standing. More information about the LANL High School Internship Program is available at https://www.lanl.gov/careers/career-options/student-internships/high-school/index.php. Information about addition opportunities are available on the STEAM Hub of Northern New Mexico website at https://www.nmsteamhub.com/.


Q: I would like to ask Ms. Astrid Morreale more about her pathway in education. I was wondering if it was possible to join the military and the path she chose, but instead of engage in combat like she did and instead perform a STEM related service.

A:Not everything involves combat. There are many other jobs one can do where you will be guaranteed to be off the front lines and be in a STEM field: bio-chemist, lab tech, medical field, engineering etc. Also there are branches that specialize in domestic live saving missions; help citizens after a big flood/fire, medical emergencies, earthquake search and rescue etc etc Astrid Morreale

Q: How would our society be now if the women pioneers of science didn't come forward and it remained exclusive to men?

A: What a really interesting question! I don’t know that there is a specific answer, however, you can look around at what women do, what professions they are in and you see that they view the world from a different perspective than men. I think that if STEM were exclusively male than you would not see as many solutions to family and community problems.

Wednesday June 16


Q: We were more focused on the pros of exos. What about the cons of exos? Are there any issues with utilizing exos that later in life people have to deal with?

Q: How does biophysics help to study DNA?

I would say that the discovery of DNA not only helped us understand how genetic information is transmitted through generations; it also gave rise to a number of new fields, of which biophysics is one. Unlike pure biology, where experiments often involve observing live cells and understanding their peculiar behaviors without unnecessary physical manipulation, biophysics has a different approach and studies live cells from a physics perspective. For example, a biologist would ask a question— “what happens if a gene is no longer expressed, how does this affect the organism?” whereas a biophysicist would want to know if the fact that a given gene is not expressed (turned off) is affecting cell structure and other properties by say stretching it and recording associated forces and changes from a control case where a given gene is expressed (turned on).


You may find this Forbes article interesting— it also features the work Karissa did on the largest billion atom simulation of an entire gene.


https://www.google.com/amp/s/www.forbes.com/sites/evaamsen/2019/04/25/what-does-dna-look-like-after-66-years-were-still-learning-more/amp/


There are many things we still do not know about DNA (for example, we study how the DNA folds to form chromosomes— I am attaching a video of a chromosome model I made below). There is plenty of work to do for the future generation of scientists like you! -Anna Lappala

Q: (Alex Miera) Will there ever be another type of exoskeleton in the future? Since you've already created a prosthetic hand - what other type of exoskeleton have you created?

Q: How does the hopper robot jump that high? Also how does it know when too jump and where to jump

A: The hopper drives around autonomously, but the jumping portion is controlled by a human and instructs the hopper how high to jump by setting the angle of actuators and force of spring mechanisms. C.Gilbert

Q: Question for Athena- Are the truck drivers that transport the nuclear power trained to work with it in case there is ever an emergency?

Q: I actually have a question about how the younger kids can possibly take advantage of the camps resources later in life.


Tuesday June 15

Q: If too much radiation causes cancer, why is radiation used in cancer treatments? Are different types of cancer more sensitive to radiation?

A: Great question! All radiation is not equal. Radioisotopes used for medicine usually have very short half-lives (the time it takes for half of the isotope to decay away), usually hours to days. This is important so there is the maximum impact on the diseased cells but effect on the body as a whole is minimized. In addition, the delivery method for the isotope is essential. Isotopes can be targeted by tethering them to a biological molecule like a protein that is attracted to the tumor, so the decay energy goes to a specific place. The type of radioactive decay is also important. Alpha emitters are good for small tumors (short path length) and beta emitters for larger tumors (slightly longer decay path). All of these different aspects have to be considered and optimized when designing a radioisotope therapy. Eva Birnbaum

Q: Question for Heather- What was the most difficult part when you were apart of the launch team for perseverance?

Q: My question goes out to Marcey Hoover: You mentioned your passion to leave the environment a little better than you found it. I was wondering a few examples of what you do in your personal and professional life to fulfill this urge.

A: On a professional front, I take the time to speak at both internal and external events, encouraging people to take on careers that take action to combat climate change and contribute via science and engineering. Here is a link to a panel session that I moderated earlier this spring form the DOE YouTube channel:

https://www.youtube.com/watch?v=w7OA8MljFqw

On a personal front, we do simple things. For example, my family opts to take our car that gets better gas mileage on any driving trips to not use as much fossil fuels, we only heat and cool the portions of our house that are used constantly (e.g. kitchen and bedroom) to conserve energy, and we have planted low-water use landscaping at our house to conserve water!

Q: What do you think you will be working on in the coming years (as your job appears to "change throughout the project lifestyle")?

Q: I would like to know what factors were taken into consideration when developing the vaccine and how it would mimic the virus.

A: The main two considerations were safety and efficacy.

The concept is as follows: the goal is to expose a person to a small piece of the COVID-19 virus enabling the person's immune system to specifically recognize the full COVID-19 virus; however this piece can't be too big (too much of the virus particle), otherwise this could result it infection. In addition, the synthetic piece of the virus needs to be stable enough to last in the body long enough to produce a robust immune response. The spike protein fits the bill perfectly. The spike protein resides on the outside of the virus and is the main piece responsible for interacting with our immune systems. The spike protein by itself connect cause an infection. It also is quite stable.


In terms of the Moderna and Pfizer mRNA vaccines, these vaccines consist of the mRNA surrounded by a shell. The shell and the mRNA need to be designed to be stable enough to survive the trip into our bodies' cells to the ribosomes, which will use the mRNA to make the spike proteins. Because the mRNA only codes for the spike protein, it cannot cause an infection. - Karissa Sanbonmatsu

Q: We’re all rovers tested in chambers or just some?

Q: For Amy Tainter- Do you think working so hard on the ranch growing up and in your adult life has helped you in your career? If so, in what way?

A: I can honestly say that most of my work ethics came from my two hard working parents (do not have an education) but worked so hard for my brothers and I to have everything we needed. second came from raising animals where my dad taught me discipline, responsibility since 11 years old and dedication to my projects (steers, swine, lambs, and horses) year after year. Then I have to thank my FFA advisor in High school who also disciplined, taught me all about life and first and for most to never be late and how to speak in front of a crowd. Ranching is only a small fraction of hard work it takes dedication, perseverance and goals/drive to want it because it is not an easy life style.

Sometimes as a woman in any career whether it be ranching, Engineering, Physicists, chemist, nursing, doctor, teacher, or officer you might feel that you have to prove yourself at one point or another but at the end of the day, the only one you have to prove to is yourself that you give everything you do 100% and you love what you do!- Amy Tainter

Monday June 14

Q: What happens to the solar panels if it hails outside?

A: Part of designing and manufacturing any product is to build into the design features to protect it while it is operating. This is what many engineers do in all types of industries including solar. Additionally, governments (state and federal) can sometimes write laws that say products must survive in certain conditions: think buildings in earthquakes and hurricanes. IN which case, a team of engineers will design and execute tests to make sure their product meets the regulations. I was curious about hail on panels too because my family is going to have them put on our house and we get a lot of hail. So I found this information from the Dept of Energy https://www.energy.gov/eere/articles/hail-no-national-labs-solar-panels-survive-severe-storm Looks like we should be ok! M. Decroix


Q: Do you trust the vaccine even though it wasn't tested for 10-15 years, and it was tested on humans?

A: Yes I do because the science behind it is solid. very often when we start a project we do our homework thoroughly so there is a high chance of success. With Covid the pressure is a lot to get it right the first time - there is no scope for mistakes and the learning curve by default had to be steep. A lot of brilliant minds and well-intentioned people were behind the vaccine. Also, remember that technology now allows to do a lot in a very short time. So it isn't like the old days where testing was much slower. Also, approvals take forever but in the case of the vaccine all hands were on board - we did not have just a couple of people at the FDA look at the application snail-pace but we had the technology scrutinized at multiple levels in parallel. Further, the mRNA technology used to make the vaccine has been in the pipeline since a couple of decades, scientists have been working on it for a long time but there was no immediate need for developing a new vaccine with this technology. Now the need is there and the technology was available - no reason not to use it.


Hope that reassures you about this vaccine. -Rashi Iyer

Q: How can you trust the vaccine if there was no critical trials on it and it was only made in a couple of months and most shots/vaccines are made 10-15 years after they start working on it, and how do you know your not going to get sick from taking the vaccine, because the flu shot for example, people got the flu shot and they still got the flu. There was a guy he got the vaccine and a couple weeks later he got covid and he died.

A: Sandra Begay: Here's the information I trust to find out details about the COVID vaccine:: https://www.cdc.gov/coronavirus/2019-ncov/vaccines/keythingstoknow.html?s_cid=10493:cdc%20covid%20vaccine:sem.ga:p:RG:GM:gen:PTN:FY21


Q: Do you often face failures in your work, if so what are some examples, and how did you and your team overcome them?

A: LOL the work is wrought with challenges and failures. But the failures teach us a lot more than the successes - we have to trouble-shoot endlessly and we learn a lot during that process. We often have heated discussions about the correct approach and we don't always agree. The buck often ends with me and I have to make the final decision. I do so after much deliberation with everyone on the team - right from the undergraduate student who does a lot of the testing work to the senior scientist with the expertise. Often what we believe in theory should happen does not work out experimentally - so input from the students and postdocs is critical in the decision making. For eg., it was my idea to use hollow fibers for the lung but my graduate student and postdoc who grew the cells in the fiber and then performed the testing could not get it to work. we kept at it for nearly 6 months while we tried other alternative approaches. Finally, I pulled the plug on it. We make small and big decisions everyday that impacts the outcome of our work. I have been very fortunate that we have a great working team with few egos and a lot of ideas. In the end we all have the same goal. Rashi Iyer


Q: Are there gonna be more help sessions for the rover?

Yes! absolutely. Let's say from 5:30-7:30 for the next two evenings. That will give you a break and you can go get some dinner. Also the mentors usually meet about 4:30 to discuss how the sessions went. I may need to send a separate google meet link - let me talk to the other mentors and see how we're going to arrange a room and I'll post that information here and also send it out to the email list. - Matt

Q: Could we code from our own personal computers or do we need to use the Pi Top?

You can use your own laptop, it is probably recommended. Here's how to download the Arduino IDE onto your laptop.

Go to: http://chamisaelementary.com/phys_camp/home.html

and you will see links for the Arduino IDE for Windows and Mac. The Windows link takes you to the App store and installs it. The Mac link puts a file in your Downloads directory and when you click on it, it should install. (I don't know Macs, but I think that is right).

Installing the QTRSensors library is exactly the same as on the Pi Top.

-Matt

Q: What is the best advice Rita Gonzales would give to incoming internship students when it comes to succeeding but also taking care of themselves?


Friday June 11

Q: How do I secure my network?

Answer by Caren Shiozaki:

1. I always change default login passwords and user names (these are what manufacturers assign so devices are ready to go "out of the box")

2. When I get a new device, I remove the bloatware (software and apps that the manufacturer automatically installs, but not that useful).

3. Strengthen security on the wireless router: use strongest encryption protocol available; as in (1) change the router's default admin password;

change the default SSID; disable WPS (wi-fi protected setup)... which allows a wireless device to join a network w/o entering the network password;

keep firmware on my router up-to-date.

4. I keep operating system updates/patches up to date on my devices.

5. I backup my data regularly. I have a local separate device that is not networked, and I also have a backup in the cloud.

6. I run up-to-date anti virus software.

7. I use a VPN.

8. I use a password manager. This makes it easy to follow best practices of not reusing passwords, and having them sufficiently complex.

9. I do not use any digital assistants (like Alexa); I won't install RING, I won't use NEST. I have a basic alarm system; if I wanted one with visual capability,

I would configure my own using servers and webcams I control.

10. The only social media I am active on is LinkedIn, and even then I am very careful about what I post. I don't use FB, Twitter, etc. A lot of this stems

from the work I've had to do with the FBI and other government agencies, and because I am called to testify or be deposed in litigation.


Q: How do you think solar panels can benefit not only the planet but humans?

A: You could say that benefitting the planet is benefitting humans. 😊 But there are other benefits for reducing our use of carbon fuels such as reductions in rates of skin cancer and fewer strong storms that damage our homes. Solar isn’t the answer for every place on the planet, but it is really great alternative source of energy for places like New Mexico. The increasing number of alternative energy sources is really exciting for engineers because now we can match the energy source to the need and location instead of always trying to pipe in fossil fuels.M. Decroix


Q: How much solar energy is needed to light up and entire neighborhood?

A:The amount of energy needed is determined by the number of energy consuming “things” in the neighborhood. Here is a neat little info sheet for street lights. https://hypertextbook.com/facts/2004/MarinaAvetisyan.shtml

If we assume every block has 10 street lights, then the amount of energy used is 10 times the energy of one street light (let’s assume 300Watts) so 3000 Watts for every block. If your neighborhood is 10 blocks, then the street lights use 30,000 Watts. It adds up very quickly. The invention of large LED lights was a huge advancement because they require much less energy, ~100 Watts, so the same number of lights uses only 1/3 as much energy. BTW, this energy can be supplied by carbon fuels or solar or other forms of energy production. M. Decroix


Q:Ms. Anna, I was wondering if you could go over what will be included with the technology package when we choose it instead of the $300 stipend.

A: All information is on website

Q: How long does it take to destroy plutonium?

A: This question of the disposition of nuclear materials like Pu is one of the legacies that will pass to your generation, to your grandchildren, and to many generations beyond us.

  1. Natural decay by radioactive transmutation (mutating from one chemical element to another) ranges from 14 years to 81 million years depending on which of the 6 or 7 Pu isotopes we are discussing, differing by the number of neutrons in their nuclei. I’m quoting half-lives which means that after one half-life the sample contains half as many “mother atoms” as you started with. For any practical sample, you never see the last mother atom decay.

  2. As Hubert mentioned in discussions, Pu can be “burned up” by fission in a nuclear reactor or an accelerator within a couple of years. The required environment has copious, energetic neutrons “to split” the Pu nuclei into harmless chemical elements such as barium. These schemes can produce useful energy. The downsides include radioactivity in the fission products and the danger of losing control of the radioactive and fissile materials, which could then be used to proliferate “dirty (radioactive)” bombs or nuclear weapons.

  3. Even if you wait for mother atoms to decay naturally or by forced fission, you are left with “daughter atoms”—isotopes of U or Am—that can be just as radioactive or chemically toxic as the mothers. Add another 5 years of processing and storage, here. The good news is that simple chemistry can be used to separate the mothers and daughters, which is generally much more cost effective than isotope separation methods by physical means.

  4. Fusion transmutes elements, too, but I’m unaware of a fusion process for big nuclei like Pu.

  5. Think of new ways to store or isolate nuclear material! Shoot it into the sun? Inject it into the Earth’s mantle at a subduction zone?

Keep in mind that radioactive materials can be safely handled and stored if respected. They are not a death ray or a plague. You are radioactive right now, and if you had a banana for breakfast your radioactivity probably doubled this morning (potassium). My father-in-law survived the bombing of Hiroshima and lived until just this year, aged 93 years, when Covid caught up with him. Alan Hurd

Q: How can you build a professional relationship with people when you are still in the introvert stage?

A: As one introvert to another, I can offer a few of tips that have worked for me.

  1. In group settings, mentally prepare your remarks by jotting them down. A concise, well-ordered logic chain will bring notice to you that may nucleate further dialog.

  2. Strike a diplomatic tone in all engagements. That will encourage people to contact you so that you do not have to take the initiative.

  3. Be a joiner. Clubs, societies, etc. When people see that actually deliver, they will eventually see you as leadership material.-Alan Hurd

Thursday June 10

Q: Do you think we would still have to code for computers in the future?

A. Absolutely! Remember the PBJ sandwich demonstration! Computers can do amazing things. Computers can learn new things on their own. But they still need guidance from humans. Perhaps in the future we will be better able to program using voice commands, rather than typing. But programming and coding are not going away. Joan

Q: When might the next rover be sent to space? What will be it's job and when do you think it might be launched?

Q: Why would computer programming be important to learn at our age?

A: Computer programming teaches problem solving and breaking problems into logical steps. This skill is extremely useful even if you do not end up programming as part of your job. Also many jobs require technology and coding skills - is it truly a superpower. In my view the trick is to find something you are passionate and see how coding can help solving problems - many times coding just for the sake of coding is 'boring' but when solving real problems it drives you. - Thomas

A. Computers and coding are everywhere in our world, so coding is a very valuable skill to learn. And lhe younger you are when you learn, the easier it is. Toddlers can learn a new language (perhaps Russian) very easily and grow up bi-lingual. Learning a new language in High School takes years are effort (and your accent will never be as good as that toddler’s! Even Anna still has an accent). - Joan

Q: Question for Joan Lucas and Thomas Proffen: What first made you interested in pursuing a career and education in coding/computer science/programming?

A: Thomas - funny enough this happened much later. I have a PhD in Physics and in the olden days, any code we needed to analyze data we collected the PhD students and later postdoc wrote themselves. I developed an interest in scientific software writing code, being admired for my coding superpowers :) Outreach started with science outreach and it was not until my daughter was in Middle School and participated in a online computer science competition that I go into coding and technology education specifically for girls and founded Oak Ridge Computer Science Girls and that led to meeting all of you this afternoon and have a wonderful day :)

A. Joan - I took a programming course in High School as an elective, and really enjoyed it. It was a fun challenge to see if I could make the computer do what I wanted. It is like solving a puzzle or playing a game. Then I took another course, and then another…. and here I am! I never was able to answer the question “where do you want to be in ten years?”. I think life rarely is something that you can carefully plan out. You just follow your passion, keep your mind open to new experiences, and see what happens!

Wednesday June 9

Q: Why are wavelengths so important?

A: The wavelength of light tells you how much energy a photon has. There’s a famous equation:

Energy = h times c/wavelength = hc/λ

where:

E is photon energy (Joules),

λ is the photon's wavelength (meters),

c is the speed of light in vacuum - 3x108 meters per second

h is the Planck constant - 6.62606957 × 10−34 (m2kgs−1)

Remember, the definition of Energy is ‘the ability to do work. The unit of energy is a ‘Joule’, which means the work done by a force of one ‘newton’ acting through a distance of one meter. One ‘newton’ is equal to the force that that gives 1 kilogram of material an acceleration of 1 meter/second/second.

All these unit definitions can be confusing, but just remember, light can do work, whether it is moving the vanes in a Crook’s radiometer, or heating up a solar panel to create electricity. The wavelength of light can ultimately tell us how much work that light can do.

Another reason wavelengths are so important is that they tell us what kind of light we are dealing with. For example, wavelengths from about 400 to 750 nanometers are in the range of visible light. X-rays have wavelengths from about 0.01 to 10 nanometers.


Q: Ms. Casperson, on one of your resume slides you had mentioned that it’s best to not use relatives and references on a resume, but would it be okay to list a relative that was a past employer? Or is it best to avoid listing relatives (past employer or not) all together?

A: It is best to avoid listing a relative as a reference because their reference will typically be viewed as biased in favor of the family member. That said, you should still list the employment in your resume and you can certainly discuss the position in your interview.

Q: Ms. Lebak, I’m not sure if you mentioned it during the lunch break, but are there any internship opportunities at N3B? If not are there any volunteer opportunities?

Q: If someone did have a non-professional resume, what is somethings that would still help them qualify for the opportunity? A: Lots of activities other than jobs can still give you relevant experience for a job posting. Just a few examples: internships, volunteering, participation in extracurricular school activities and clubs (like theater, Best Buddies, the GSA, etc.), participation in sports while still maintaining a high GPA, advanced coursework (AP, IB, etc.). Indeed.com has some suggestions for writing a resume with no experience here. - Sara M.

Q: With cybersecurity, how can we block scam callers on our phones?

A: There’s 2 ways. The best is to sign up for the National Do Not Call Registry https://www.donotcall.gov/ Also a lot of devices have the ability to manually block phone numbers.-Olivia Stella


Q:What are the requirements to become an ESA astronaut? What classes should someone take in college to become an astronaut?

A: The requirements are surprisingly simple! You need to:

  • be between 18 and 50 years of age

  • be a national of one of the ESA member states: Austria, Belgium, Czech Republic, Denmark, Estonia, Finland, France, Germany, Greece, Hungary, Ireland, Italy, Luxembourg, The Netherlands, Norway, Poland, Portugal, Romania, Spain, Sweden, Switzerland and the United Kingdom or 'partner states': Canada, Latvia, Slovenia, and Lithuania

  • have at least a Master's degree in a STEM subject or Medicine

  • have 3 years of professional experience in the field (e.g. 3 years of PhD research count)

  • meet the medical criteria of a pilot (you do not need to have a pilot license but need to be physically fit to become one)


As for classes in college, essentially anything that ultimately leads to a degree in STEM - chemistry, physics, engineering or mathematics. You can also become a doctor and then be sent to space to investigate *in situ* the response of human body to 0G environment!


Q: How does the milky way galaxy account for all of its planets and star without crashing and dying out?

Even though there are about 200 billion to a trillion stars in our Milky Way, the empty space between the stars in humongous. For example, from here to the next closest star is about 20 trillion miles. So space is vast and mostly empty. Here is an example: the Andromeda galaxy is about the same size and shape as our own galaxy. It is on its way here, and will collide with our own galaxy in a billion years or so. You would think that there would be a lot of collisions between stars. But because space is so vast, there will probably not be a single crash.

(Hubert)

Another thing to think about is angular momentum. As you will learn in physics classes both linear and angular momentum are conserved in the Universe. In case you were wondering whether stars e.g. move towards the galactic centre and collide because of gravity, then this is not the case. A star stays in its orbit at a distance away from the galactic centre owing to angular momentum and does not change its course unless its host galaxy crashes with another galaxy, causing the star to gravitationally interact with other objects.


Q: What makes light exactly? Like how are the electrons in light seen by us?

Light is made by charged particles like electrons. Electrons surround the nucleus of an atom in what we call ‘energy levels’, but they don’t always stay in one level. Sometimes the atom is disturbed and an electron can move to a more energetic level. The electron wants to get down to its original less energy level, and to do so it emits a photon of light which carries the extra energy away. That is where most light comes from.

There are no electrons in the light that we see every day, just photons. Electrons do move around outside an atom (like an electric current), but they are very easily stopped when they hit something, even something as light as air. So they really never get to your eye. Astronauts in space report flashes of light from electrons and other charged particles hitting their eyes coming from the vacuum of space where there is nothing to stop them. But we don’t see this at this surface of the earth.


The answer to the second part ( how is light seen by us): The light-sensitive layer in the back of your eye is called the retina. In the retina there are light-sensitive cells. In these cells are molecules that are sensitive to photons of particular range of color (or wavelength), or to just light of any color. When a photon of light falls on these molecules, the energy of the photon is absorbed by this molecule, and this causes a whole chain reaction of chemical and electrical activity, with the result that an electrical signal is fired off from the retinal cell towards the brain. In the center of the retina there are such cells that are sensitive to 3 different colors: mostly red, mostly green and mostly blue. Cells that are sensitive to light, but not to any particular color, are sprinkled over all of the retina.



Q: What if you apply for more then 1 job, do you need to have a different reference or the same person ?

I have used the same person as a reference for different job applications, as long as that person can speak to my qualifications for that particular job. Also, it is good to let them know each time you use them as a reference . - Sara M.

Tuesday June 8

Q: Are we going to be doing mini project like every day?

A: On the first week, we are learning the concepts and doing small parts but next week every afternoon will be dedicated to working on the specific projects (MARS Robot or Ohana Hmestead)

Q: Do men feel threaten by women in science? Is it because the social norm at that time.

A: Remember that men and women are just human beings and we all have our own personalities and behaviors. Some are based in our genes and some come from our experiences. Some men will be threatened by new interactions and certainly there being more women in the workplace is new to some of them. But I have also found some women can be threatened as well. So gender stereotypes really don’t take you very far when you are on a team trying to do a job. Interactions/Relationships with people are one of the hardest parts of any profession/job. And I think, on average, more women are aware of those difficulties than most men. I try really hard to look at everyone I interact with as someone to learn about. I don’t always succeed, but I try. -Michele Decroix

Q: What does the alligator clips have that also helps light up the LED?

A: Alligator clips are made of metal, which conducts electricity, and they have a spring which makes it easy to clip onto other wires or electrical bits.

If you don't have wires with alligator clips, you have to strip the plastic insulation off some wire and wrap it around the LED legs, which is less convenient and more finicky.


Q: Do you believe Project Y would have as successful as it was without the women who contributed?

A: A: I think that the varied contributions of women were essential to making Project Y a success. The wartime laboratory at Los Alamos relied on women of science including medical staff, physicists, chemists, and mathematicians, but also needed the support of women to build a community for Project Y staff in what was then remote New Mexico. Women, like Dorothy McKibbin and Charlotte Serber, were in charge of important administrative offices in Santa Fe and in Los Alamos. Women in the military (the WACs) stepped in to take over previously male-only wartime roles, working as drivers and supply clerks. Local women from the Pueblos and nearby Hispanic villages worked in the technical areas and in laboratories, where their fine motor skills were essential in the assembly of small-scale experimental devices that led to the development of the Fat Man weapon. Women, like Frances Dunne and Norma Gross, made valuable contributions while working in hazardous field conditions. Without a doubt, women were key to the success of Project Y and to ending World War II, along with women working across the country at other Manhattan Project sites, such as Oak Ridge, TN; Hanford, WA; and New York’s Columbia University.-Ellen McGehee

A: Fun fact: There were no women engineers on Project Y. Engineering was not a common thing for women to do and still remains less common then women in science. Frances Dunne and Norma Gross did engineering work but they had science degrees. There was one woman, Miriam White Campbell, who joined the Women’s Army Corp (WACs) and worked as a draftsman for the project. She drew the final drawings to assemble the Little Boy weapon. Her work was engineering. Today we would call her a designer and she would probably have a mechanical engineering degree or a mechanical technologist credential. If she were working today, she would probably be doing a lot of 3d printing! There is an interview on You Tube where she talks about bringing her dog to live with her in Los Alamos. https://www.youtube.com/watch?v=R5Q0RP3x5pc -Michele


Q: How exactly does the copper tape work.

Materials that allow electricity to flow through it are called conductors. In this group are all metals, like copper, iron, aluminum etc. Also water, and all wet things, such as your body, potatoes, lemons etc conduct electricity.

Materials that do not allow electricity to pass are called insulators. In this class are plastic, rubber, cloth, paper, dry wood, air etc.

This is why for example extension cords are copper on the inside, and plastic on the outside. You can touch the cord and not get hurt, and why you don't want to touch the prongs of a plug while it is mostly plugged in.

Our copper tape is made of metal, which is a conductor, so electricity flows through it, while the paper we tape it down on does not. (Our particular copper tape has glue on it that is also made to conduct). So it is like using copper wires with a plastic insulation on it.

In the experiments we do in the camp, voltages are low enough that they are safe: you can touch the wires safely. The limit for this kind of safe work is 50 Volts. (the highest voltage we use is 9V)

(Hubert)


Q: Olivia stella: if you link multiple social media accounts, like linking spotify to instagram and instagram to apple, or something like that, does it increase the risk of you getting hacked?

A: Yes, because if they get your apple password, they have access to everything else that uses the apple password. And even if it’s just social media accounts, they might be able to use the information to access more sensitive accounts (shopping accounts, bank, etc.) Here’s a good article from 2014 (yes, it’s a persistent problem) - https://www.pcmag.com/news/reusing-passwords-across-social-media-sites-dont-do-that


Q: What makes the motor we made today with the copper wire,battery, magnets and paper clips spin ?

Remember that an electrical current running through a wire produces a magnetic field. In our motor we made, this effect stronger by winding the same wire around 7 or 8 times - this makes the magnetic field stronger. The next thing to remember is that magnets can push or pull on each other. In the little motor we made, the parts are arranged such that the magnet at the bottom pushes and pulls on the wire-coil-magnet (when we send a current through) to make it spin.


Q: I forgot what the 3 things you need to make electricity was called so my main question is what were the 3 components needed to make electricity?

If the question refers to the potato battery, this is what you need: first you need 2 different metals, plus some fluid in between, like the juice in the potato, or orange juice, or even salt water.


Q: Olivia Stella: discussing about cybersecurity " will having similar passwords on multiple platforms increase the chances of being hacked?" This seems like an important thing to know to safeguard ourselves. Other than the lessons though, I was wondering if we would have a lecturer talk to us about medicine as I currently want to study medicine and it would be really cool to learn/explore pathways for it.

A: Yes, because if they get your apple password, they have access to everything else that uses the apple password. And even if it’s just social media accounts, they might be able to use the information to access more sensitive accounts (shopping accounts, bank, etc.). Here’s a good article from 2014 (yes, it’s a persistent problem) - https://www.pcmag.com/news/reusing-passwords-across-social-media-sites-dont-do-that- Olivia Stella

Monday June 7


Q: Why are women so underestimated in the science and engineering field?

A: Underestimated is an interesting word because it depends on your perspective. I think that the field of engineering has been so male for so long that it has just taken on the behaviors and thought processes of men. It is so normal now that no one really even thinks much about how it could be different. Right now the profession is only about 20% female at the entry level so we are having a tough time breaking through with our thought processes. Areas of engineering that are changing fastest are Bio-engineering and energy/environmental because they have a higher number of women in the fields. My area of Mechanical and Aerospace is still less than 15% female. I do feel underestimated sometimes but I am not sure it is a conscious thing most of the time. I believe I just solve problems differently and it seems abnormal to my colleagues. It is frustrating! It has been helpful for me to raise the consciousness of my male colleagues about how they solve a problem. I ask a lot of questions.😊 BTW, this is true in reverse as well. My mom was a nurse which is traditionally a female profession. When male nurses started being more “normal” there were a lot of the same discussions in that field. So it is really is about being the minority in a field and not always gender.-Michele Decroix

A: Because people are weak, and it has always been easier to settle for a stereotype (as in gender roles and racism) than making

up our own mind. Unfortunately, these views can trickle down the centuries if they are not being questioned. But don’t assume that you will have a bad experience in STEM! There are strong efforts all over the world to change this situation.-Marein Rahn

Q: Why do some materials work better with different colors of led lights?

A: Every material in a circuit has a property called impedance and the value for impedance represents how easy or hard it is for electricity to “flow” through the material. Copper has a pretty low impedance and conducts electricity well. Other conductors have higher impedance and conduct electricity poorly. So if you use a different material with a different impedance in the same circuit, then you get different voltage and current output and you may not be able to light the same LED. Electrical engineers must choose materials carefully to get the right performance from circuits without “wasting” a lot of the energy, because the electricity that is not conducted through a material is lost energy. The “lost” energy is stored in the material which causes it to heat up and the components of the circuit can get very hot. Many electrical components that handle high power (like transformers in the electrical grid) need to be cooled, usually with oil. There is a lot of research happening in materials to develop broadly useable superconductor materials that can handle high power with very little loss in heat. This is a tough problem because the materials also need to be mechanically strong to withstand weather and tension between the poles and other design constraints. But if we can develop the right materials, then we can reduce the losses in the national electrical grid. This will allow us to move electricity around the nation further and faster without wasting energy.

Q: How do you get to be so determined about what you want to do so early?

A: In my case, I found something I was interested in at around age 14 (learning languages) and just kept looking for ways to keep doing that, eventually ending up with a PhD in Spanish linguistics. I never stopped liking it and still find it fascinating. However, it is not common to find what you want to major in and work on professionally when you are 14. If you aren't sure about that, there's no reason to feel bad or like you're falling behind. Also, even if you find what you want to major in and work on that early, it can still change later on (like it did for me). It is normal to have several different careers over the course of your life. - Sara Mason

A: I was not determined about what I wanted to do when I was your age. For some reason I wanted to be different than my sister. She liked science but was more interested in the medicine/biology side. I liked physics because I had to think through the problem. It wasn’t just memorizing things. Also, I could make things happen in the real world, like throwing a ball or shooting a projectile, and see the physical laws in action (gravity). My sister and I took physics class together and I was better at then she was. That was very frustrating for her. A little competition is healthy I think. 😊 My dad was a skilled tradesman in construction and worked with a lot of engineers. He said, “You should be an engineer”. I said ok but I did not know what it was. I just knew it paid well. I almost dropped out my sophomore year in college because it was really hard. But I stuck it out and the next semester I took a class in thermodynamics and found my passion for studying fluids. From there I went on to work on designing airplanes and doing research in combustion. Life opens lots of doors. The trick is sticking around so you can walk through them when you are ready.-Michele Decroix

A: I think the biggest challenge nowadays is that there is so much stuff that catches our attention at the same time, so it gets very difficult to focus on one thing. The problem is that to develop a passion for something, you really have to get into it first. So my advice would be just to select one thing (really, anything. On this page alone I see several questions that could ignite and support a whole career) that you feel you care about, and start digging into it. Read a book, sign up for a course, set yourself some goal, start a project with friends … . What’s important is that you keep digging for a bit, even if it takes some effort. The good news is that most topics get more interesting, the deeper you dig. And of course you can adjust the direction of your dig (just don’t jump to the next hole at the first obstacle!). What you have to look for is your "positive feedback loop”: Then the more you learn, the more fun it will be and the more successful you will be, which will motivate you more and so on. It just has to come from yourself (teachers and professors can only nudge you in a direction, but the initiative must come from yourself).- Marein Rahn

Q: How does electricity produce from metals and a potato?



Q: What is the weakest build of an atom? Is it graphite?

Q: How do crystals begin to form, and how does it keep its solid state?

Q:What happens if a crystals was to become liquid? Or has that never happened?

Q: What academic advice do you have for someone who wants to pursue a career in software architecture?

A: The field of Computer Science is incredibly broad, including circuit design, programming languages, Artificial Intelligence, Algorithms, Networking, etc. You will probably want to get a degree in Software Engineering, rather than Computer Science. But you don’t have to decide right away. During your first two years in college (freshman and sophomore) you should take broad range of courses to see which ones really are interesting for you. You cannot take too many Math courses! Mathematics, and that kind of logical thinking, suffuses every aspect of computing and also science!- Joan Lucas

Q:What advice would you give your younger self if you could time travel?

A: My advice for someone going to college would be to ask any question that you have, right away (if you leave them for later, they will just pile up). Don’t worry about what others think and be confident that every question you have is justified. -Marein Rahn

Q: What do zinc and cooper have in common to light up an LED light?

A: What zinc and copper have in common is that they are both metals, and metals conduct electricity very well. Because they are different metals, they react differently with the fluid in the potato, and this gives rise to a voltage difference between the two. See the video on batteries in Pascale's presentation on Wednesday:

https://youtu.be/9OVtk6G2TnQ -Hubert


Q: I wonder what is something Marie always followed and told herself when choosing to keep pushing forward not only for herself, children but also her work and studies to benefit even after she passed?

A: Marein: It’s a difficult question, since Marie wasn’t the kind of person who wanted to share too much of her personal life with the public. Maybe her drive was so strong because she was able two align two goals in her life: On the one hand, her work gave her an immense personal satisfaction, and on the other hand she grew up with the humanitarian conviction that all of us, individually, have a duty to improve the world. I think few people get to combine their own goals and life philosophy in this way. Either because they never get the chance, or because their personal desires are not compatible with a fulfilling cause (e.g. if your only goal is to get rich, you won’t have much to keep you going if you fail). Also, I think most of us often deceive ourselves into evading challenges, mostly for irrational reasons. E.g. sometimes if you are “scared" of an exam, you don’t want to study for it. I think Marie’s had such a logical and scientific attitude that she just didn’t accept obstacles as easily.


Another amazing story I didn’t get to mention:


Marie really didn’t enjoy the public attention she received in her later years (after the Nobel prizes), not even that of the early feminists of the time. But the editor of the US women’s magazine The Delineator, Missy Meloney, was so insistent that she started a campaign to fulfil Marie’s greatest wish. In the 20s, radium was already industrially produced, but Marie couldn’t afford any for her research (one gram cost 100.000 $, which would be more than 1.3 m. $ today). Mrs. Meloney managed to raise this money from her readers. So Marie agreed to write the “The Story of my Life” for the delineator, for "the encouragement to the women of America”. Together with Irene and Eve, Marie then travelled to Washington D.C. and was handed one gram of radium in a lead-lined mahogany box from president W. G. Harding. I have uploaded the relevant issues of the Delineator here: https://drive.google.com/open?id=1pgNCUT8hZSRzyuvL2samX9_YjgGpzFMm.


Q: Ask Frances Chadwick: What is the most challenging part of your job?

A: the span. Trying to get a communication just right for 13,000 people is hard. And you can’t please everyone. You’re trying to take into account all the stakeholders (often including external as well), all the reactions, often with a deadline or some other pressure. And working with the federal government – well, it’s very bureaucratic, so that can sometimes add frustrations.

QI Was also wondering about the statistics of females in medicine in relation to the data shown in both Anna'a brief presentation and Irene Qualter's data.

Q:Could there ever be a technological development that would suppress radiation?

A: Good question! We can only really “suppress” radioactivity by shielding it. Atoms are radioactive when their cores (nuclei) are too unstable to hold together. The cores consists of neutrons and protons, which are held together by nuclear forces. These work quite differently from other forces that you are familiar with, like graviational or magnetic ones. It’s very difficult to manipulate the world at that scale, and we will not be able to change the “half life” (= average stability) of an radioactive element. However, in science you can never say never! If you were able to "design your own nucleus” (e.g. by adding neutrons), you might indeed make an unstable atom stable. Indeed, theorists have predicted an “Island of Stability” in the (largely empty) map of possible nuclei, see https://en.wikipedia.org/wiki/Island_of_stability . It’s not clear if we will ever get there though!- Marein Rahn

Questions to Frances Chadwick:

Q: What are so things over here at New Mexico that are different from were you're from?”

A: The food! The people! And the weather! Seriously, this is a great place to live - I can imagine if you grew up here, you would take it for granted, but NM is unique, and you should always remember how lucky you are. Another big difference is the open space/lack of population. The UK is a very crowded country - traffic everywhere, houses everywhere - its hard to get away. There are things I miss too of course - family, and the sense of history.

Q: “What was the hardest thing to adapt to?”

A: I think when you are a first generation immigrant, you always feel a bit “torn” - I love NM and working/living here, but I do also feel some nostalgia for places I knew as a child. But having said that, I have always felt very welcome here - not treated as an outsider, and I very much appreciate that.

Q: "Do you think it is important for young women to be involved in the sciences from early on?”

A: I dont' know. What I do know is that at some point - Sometimes at a young age, other times later on, I think it helps to find something that genuinely interests you. Often it is a teacher who can create this spark of interest, or a parent, or it can be something you discover on your own. I have three kids - one decided what she wanted to do when she was a junior in high school, another doesn't know what he wants to do when he grows up but is studying a subject he is deeply passionate about and has been since he was about 10, and the third has absolutely no idea and still thinks all school is boring!! So i hold out hope that in time, the third will find something that does interest her because it makes life more enjoyable, for one thing!

Q: “What was the easiest thing to adapt to?”

A: You can connect with people, no matter where they are from. I think humans are good at making those connections, if they try.

Questions from 2020 Summer Camp

Monday June 8

Q: Could people potentially go into the quantum realm and see something if they stayed in there for long enough?A: The quantum realm is not really a place you can go to. It refers to a set of conditions in which quantum effects become noticeable. Usually this refers to very small objects, like a few atoms, a system with particles of light, or super low temperatures. However, we can measure and study these (usually small) systems and try to understand how the quantum bits and pieces are behaving.

Q: What is Bragg's Law and how do you calculate it?A: When x-ray waves hit a crystal, they can reflect off each atom in the crystal. Only in certain directions do all these waves line up 'in phase', meaning that wavetops from one wave line up with wavetops from the others. These directions depend on how the atoms in the crystal are spaced. As a result, the pattern of reflected x-rays reveals the structure of the crystal. See https://en.wikipedia.org/wiki/Bragg%27s_lawAlso, tomorrow's demo on diffraction patterns address that same problem, but at optical wavelengths, rather than x-ray wavelengths
Here is the derivation of Bragg's law: https://www.youtube.com/watch?v=FRDvRhCvuHg

Q: How can you tell when an atom is radioactive? A: By definition, radioactive atoms fall apart sooner or later. The 'half-life' of any particular radioactive substance is the time in which half of the atoms have decayed. For example, if you start with an ounce of Tritium, 12 years later only half of it is left, and 12 years after that, only 1/4 ounce of tritium is left. So if you observe your starting ounce for a while, you can tell you are losing Tritium steadily. However, if I have only one atom, you can't tell when it is going to decay. So if my tritium atom is still around after 50 years, can I say it is not radioactive? Not really. It might decay one day later.Here is a cool video that talks about radioactive decay: https://youtu.be/UYvx0O8itMAor here: https://en.wikipedia.org/wiki/Radioactive_decayor here: https://chem.libretexts.org/Courses/Oregon_Institute_of_Technology/OIT%3A_CHE_201_-_General_Chemistry_I_(Anthony_and_Clark)/Unit_3%3A_Nuclei%2C_Ions%2C_and_Molecules/3.1%3A_Nuclear_Chemistry_and_Radioactive_Decay
In the lab, we can tell that a material or an object contains radioactive elements by measuring the radiation that comes out of them usually with an instrument called Geiger counter (https://en.wikipedia.org/wiki/Geiger_counter) . There are three types of radiation: alpha, beta and gamma. Alpha particles are particles formed by two protons and two neutrons kicked out of the nucleous. (https://en.wikipedia.org/wiki/Alpha_decay). Alpha particles have a very limited ability to penetrate other materials and are stopped very easily bya. piece of paper. Beta particles are high-energy, high-speed electron or positron emitted by the radioactive decay of an atomic nucleus (https://en.wikipedia.org/wiki/Beta_decay). Beta particles can penetrate a sheet of paper, but can easily be stopped by a thin sheet of either Perspex or aluminum. Gamma decay is the process by which the atoms desintegrate emmitting a gamma ray photon. (https://en.wikipedia.org/wiki/Gamma_ray#Radioactive_decay_.28gamma_decay.29)We are constantly showered by radiation called cosmic rays (https://www.youtube.com/watch?time_continue=5&v=_bKbMARsE-4&feature=emb_logo)
Did you know that several household items in your house contain radioactive sources? When you figure it out, let us know!
Brief story about Marie Curie: https://youtu.be/w6JFRi0Qm_s
Q: "Exotic magnets" were mentioned earlier, what are they? A: Here is a cool series of videos about magnetic materials and magnetism. There are 6 episodes: https://www.youtube.com/watch?v=62dez4tD5Ok&list=PLiAlc6pI18dWJT9NNRgGxJF6yHgEElwmU

Q: Do we always have to show our faces in the camera?A: Yes, except when we are on breaks. Why? because they provide feedback to us about your level of engagement and understanding. Humans we rely on facial expressions as a feedback loop in relationships. We care and we want to know you are doing well. If for whatever reason you are not comfortable with it, please send me an email and we can talk about it allobet@lanl.gov.

Q: What happens to atoms that pass through a black hole? Do they become energy? What happens to atoms when a black hole "evaporates"?A: First, let's look at what a black hole is: when people picture a black hole, you usually draw a black ball. Now this black surface is not really a hard shell, like a black billiard ball, or a thin shell like a christmas ornament. It is a boundary in space (just like the boundary between NM and AZ) from which no light comes out, but there is nothing there in that place. When an atom falls into a black hole, nothing special happens when it falls through this boundary, it just keeps on going. But once it crosses the border, it will never come out.What happens on the inside of a black hole we really don't know, except that nothing of the atom is left except it's mass (= energy, because E=mc^2) The way a black hole evaporates is that it radiates like a warm object (getting hotter and hotter, until the final little speck explodes). The radiation is a form of energy, so the black hole slowly loses energy, which means it loses mass (again via E=mc^2). So will our poor atom come out? Not really, it was destroyed long ago, and what comes out in the end is an amount of energy equivalent to the mass of the atom.
Q: How does the phase change work between martensite and austinsite? Why are the particles arranged so differently but still form martensite?A: Check it out here: https://www.youtube.com/watch?v=yR-6_lS9vtshttps://www.youtube.com/watch?v=wI-qAxKJoSU
Q: What is Bragg's Law and what is it used for?A: Check this out: https://www.youtube.com/watch?v=QHMzFUo0NL8
Q: In shape memory alloys, what allows the atoms to rearrange and why can't other materials do the same?A: In shape memory alloys, you have two states that are very close in energy. When you put some work by deforming them, you push the system in an energy state and when you add heat the whole system moves to that state and when cooled down it goes back to the lowest energy state.
Q: In a crystal, How do you know if it is made of more than one component and how would you discover what components it is made out of?A: A way to determine the crystal structure of a material is using diffraction (https://www.youtube.com/watch?v=QHMzFUo0NL8). The diffraction pattern from a material allows us to see what atoms is it formed of and what is the atomic arrangement.
Q: Why is a diamond the strongest materiel?A: Because of the rigit arrangement of the atoms and covalent bonds. check this our: https://www.youtube.com/watch?v=xJ5h9bwaaKk

Q: How many women participated in the Manhattan Project, and what was the ratio of males to females that worked on the project?A: I do not know how many women participated because women had all sorts of roles at LANL and other sister laboratories during the Manhattan project. https://www.energy.gov/women-manhattan-projectHere is a 40 min symposium about the Women in the Manhattan project: from women who worked at Oak Ridge that worked in the Manhattan Projecthttps://www.c-span.org/video/?326373-1/discussion-women-manhattan-project

Tuesday June 9


Q: Why did the magnets have to be touching the Battery to go through the coil train?A: Check out the explanation of how the magentic train works: https://www.youtube.com/watch?v=BWuICjRU3t0https://www.youtube.com/watch?v=9k7zywli4Vg

Q: If you used a battery with a higher volt would it go through the magnetic train faster or would it just last longer?

Q: How does the battery get hot when there is magnets on it?A: It is not the fact that there is a magnet. When current flows through a conductor, the conductor gets hot. That's why the incandescent lights emit light. They get so hot when you force current through them that they glow. The battery gets hot because you by making the magnets touch the copper wire when you use the electric train, a circuit is created and current flows without much resistence between the + of the battery to the - of the battery. In most uses of batteries, we don't draw su much current and batteries do not heat up much and that's why they last longer than during our experiments.

Q: What is the most radioactive element known as of present day? A: Because it is a naturally-occurring element that releases a huge amount of energy, many sources cite polonium as the most radioactive element. Polonium is so radioactive it glows blue, which is caused by excitation of the gas particles by radiation. A single milligram of polonium emits as many alpha particles as 5 grams of radium. It decays to release energy at the rate of 140W/g. The decay rate is too high that it can raise the temperature of a half gram sample of polonium to over 500°C and subject you to a contact gamma-ray dose rate of 0.012 Gy/h, which is more than enough radiation to kill you.But it really depends by the definition of "most radioactive" for example: francium — its most stable isotope has half-life of 22 minutes. But precisely because francium is so radioactive, nobody ever made enough of it to even see. (If a solid lump of francium somehow came into existence, it would instantly vaporize from its own heat.)

Q: What makes a potato a good conductor? Is it the water or starch content?A: https://www.youtube.com/watch?v=GZiLVqGd9JY

Q: How old do you have to be in order to have an internship at the lab?A: https://www.lanl.gov/careers/career-options/student-internships/high-school/index.phpStudents who wish to participate in the High School Internship Program must meet the following requirements:-Have senior standing AND be at least 16 years of age at the time the internship begins.-Have and maintain a minimum cumulative GPA of 3.0 on a 4.0 scale-Attend a high school located in northern New Mexico-Pass a new employment drug text: The Laboratory requires successful applicants to complete a new employment drug test and maintains a substance abuse policy that includes random drug testing.
Q: What do each of the dials mean on the multimeter? I would like to learn more about voltage and what it means. A: Different multimiters have different readings possible. Typically they measure electric Voltage (V). Voltage is the difference in electric potential between two points. If volatge is 0, there will be no current flowing in a conductor. Example: take a copper wire and measure the voltage between the two ends, it should read 0V. But when you measure the voltage across a battery, you should measure a value different than 0 if the battery is not dead. AAA batteries shoudl read 1.5V when new.Another thing multimeters measure is electrical resistance which is often refered by the greek letter Omega (Ω.). Electrical resistence is how difficult for the electrons to flow through the material. Eg: if you take 2 pieces of a wire of different lengths, the electrical resistence will be proportional to the length of the wire. The electrical resistence depends on the type of material, the length of the material and the diameter of the material.Another thing a multimeter can measure if the current or electrical intensity that flows through a wire. It is measured in A (Ampere or Amp)Ohms law says that the intensity in a conductor is equal to the voltage across the conductor divided by the resistance of the conductor.Check it out! grab your resistor and conect it to a battery. Measure the Voltage at the battery first. Connect the leads of the resistor to the battery and measure the intensity flowing in the system. Now do the math and check if the resistance calculated is equal to the resistance in the label of the resistor!Multimeters There are two types of Voltage and Alternating Volatge (AC) and Direct voltage (DC). Batteries generate DC Voltage. Alternating voltage is when the voltage is delivered in sinusoidal form and it is often refered with a wigle on top of the sign for Voltage. The voltage received in households is AC at 60Hz so 60 wigles a second. [NEVER USE A VOLTMETER ON AN OUTLET OR ANYTHING PLUGGED TO THE WALL OUTLET]
https://en.wikipedia.org/wiki/Multimeterhttps://en.wikipedia.org/wiki/Ohm%27s_law
Q: When sphagettification occurs near a black hole, what causes the black hole to have more of a pull on your feet in comparison to your head?A by Kaisa: A: Gravity works in the sense that the farther away you get, the less it pulls. While standing on earth's surface, you have more gravitational force pulling on you than if you were to be in space, and black holes are just an extreme example of this. Because the gravitational pull from the black hole is so extreme, it just takes less distance for this effect to happen, resulting in spaghettification.
Q: Is there a device that can actively see and/or measure the magnetic field around an object?A: It depends on the strength of the field. The device is called a Gaussmeter or a magnetometer. I would typicall use a gaussmeter to measure the magnetic field around an object while I would use a magnetometer to measure the magnetic field created by a small sample. Gauss is one of the units of magnetic field measurement. It is characteristic of small fields while Tesla is another unit that represents larger fields https://en.wikipedia.org/wiki/Magnetometerhttps://en.wikipedia.org/wiki/Earth%27s_magnetic_fieldOne tesla (T) is equal to 104 gauss
Q: Regarding the issue about cooling the wires, why not test magnets in an already cool environment, or at least a controlled environment where we can put the room(?) into extremely cold temperatures?A: To avoid magnets from heating up, they are often submerged in liquid nitrogen and therefore at 77K. However, the larger the magnetic field one needs, the higher the amount of current needed and the more massive the coils are. If the coils are large, then there is a challenge of thermal transport between the inside heating up and the liquid nitrogen not being close enough to cool it down quickly. Remember resistence in a metal goes up as temperature raises so the more the coils heat it, the more resistent the coils are and the more resistent thet are the higher they heat up. To avoid this. scientists have optimized paths for the Liquid nitrogent (or coolant) to keep the coils as cold as possible througout the thickness of the coil.
This about this: if you want to cool a glass with water, how do you think it will cool faster: 1) if you put ice around the glass or 2) if you put ice around the glass and ice inside the liquid?
Q: What is the difference between a lowercase m and an uppercase M on a multi meter? A: small m typically stands for mili, or a thousand. of the unit. So 1 mV is a throusand of a Volt or 0.001 V. M typically stands for Mega. It stands for 1,000,000 of the unit. So 1 MV is 1000mmmVhttps://en.wikipedia.org/wiki/International_System_of_Units

Wednesday June 10

Q: Why is wavelength important in spectrophotometric measurements?A: One way to figure this out is to use small electrical probes in the eyes or brains of say a shrimp and listen for electrical activity of brain cells when you shine ligh of different colors. But there are easier ways. For example, we know that certain flowers have parts that reflect or absorb UV light very strongly. We can't see these markings, but some insects can. So you make some fake flowers, some with these UV patterns, some without. To us they would look all identical. But then you notice that the insects only seek out the ones with the UV markings. So you conclude that insects can tell the difference, and therefor can see UV. BTW, human eyes have 3 different color sensors (red, green, blue), but mantis shrimp I believe have 16. Some people have only 2, and are colorblind, and apparently there are some people who have 4, and they can distinguis colors that no one else can.Asia: Let us answer this question from a couple of angles. Wavelength is important in spectrophotometric measurements because:A spectroscopic measurement is, by construction, a measurement of radiation intensity as a function of wavelength. Hence wavelength is the independent variable in your spectroscopic measurement.Electrons in every atom have discrete energy levels, which result in a characteristic emission line pattern (i.e. as electrons cascade down they emit light associated with discrete energies, which translate to discrete wavelengths). A simplified graph of the Hydrogen atom emission/absorption spectrum in the range of visible light can be found here: https://www.thoughtco.com/definition-of-balmer-series-604381Not all measurements as a function of wavelength can be made with the same technology. Hence, for every wavelength range there is a unique detector designed to deal with this range. For example, space telescopes which collect data in X-rays and Optical/Ultraviolet (UV) simultaneously, require two separate detectors in sync with each other (e.g. XMM Newton X-ray detector https://www.cosmos.esa.int/web/xmm-newton/technical-details-epic and optical/UV monitor https://www.cosmos.esa.int/web/xmm-newton/technical-details-om). Different wavelength ranges also differ in how difficult it is to capture and measure the energy/wavelength of a given photon. For e.g. it is much technologically easier to measure a spectrum in the visible (optical) wavelengths than in the very high energy range like gamma rays.

Q: how do we know that other animals can see UV and other spectrums of light?A by Kaisa: A: Eyes are made up of rods and cones. Scientists have found that some species, such as birds or certain types of shrimp, have more cones than humans, allowing them to see more colors than humans. (Fun fact: magenta is the color that the human mind makes up because we cannot comprehend what the color actually looks like)
Q: Why can't we see other spectrums of light?A: Normal human eyes have cells (in the retina, which is in the back of your eye) that respond to colored light. One type reponds strongly to red, another type to green, and the third to blue. If infrared light falls on them, nothing happens. Same for UV. One reason why all creature's eyes are all sensitive in roughly the same region of the spectrum is that it is the color band where the Sun is the brightest. When you're designing an eyeball, you might as well choose to sit in the spectrum where there is the most light. If we had x-ray eyes, we would all be blind, since there is no x-ray light on the Earth's surface.
Q: what is the theory of star?A by Kaisa: A: Stars are initially formed by small particles (hydrogen; the lightest element) pulling on each other and when enough particles are present, the gravitational pull intensifies enough that the particles start to fuse. These reactions are what causes a star to create the energy to glow. What happens in the rest of the star cycle depends on the size of the star. If it is a smaller size, like that of our sun, eventually it will run out of hydrogen atoms to fuse and begin to become hotter and denser allowing helium atoms to fuse (the next lightest element). When it moves to fusing larger elements, it produces more energy, causing the star to get bigger and eventually so big the star explodes, leaving a white dwarf star in its wake. If the star is very large, the gravitational force will become so extreme that the star collapses in on itself, forming a black hole.
A: A star’s history begins whenever the local gas pressure (resulting from motion of gas particles – e.g. Hydrogen or Helium) is overcome by gravitational attraction of the gas particles (resulting from their mass). The gas then collapses into a small region and becomes packed so closely together that the compression causes first nuclear reactions to occur. That is the point at which the star ignites and is ‘born’. As time passes the star will keep on shining and gradually burn up its fuel. Once enough of it is burnt, depending on its mass, the star can swell up or explode, leaving a different form of star behind. This page explains different evolutionary paths of stars in more detail: https://astronomy.swin.edu.au/cosmos/S/Stellar+EvolutionAs correctly answered by Kaisa, the very first stars in the Universe were composed only from Hydrogen and Helium in the primordial (the way Universe started off initially) composition of roughly 25% Helium and 75% Hydrogen. As the stellar evolution progresses elements heavier than helium are formed in the stellar cores and in explosions, while the explosions deliver all the different elements into the surrounding space. As time progresses the enriched gas then collapses to become a new generation of stars and planets, including our own Earth. Interesting fact – the basic building block of life, Carbon, is the end result of Helium burning in the stellar cores. Without stars, there would be no Carbon or, in fact, other heavier elements. Therefore, one indeed can say that *we are made of Stardust*. [Asia]
Q: On an open field, how far can a laser beam (like the ones used today) be seen? Do the wavelengths of the different colors affect the distance?A: This depends on the air quality, the power of the laser, and the color. Let's look at color first: it has to do with the same phenomenon that causes the blue sky. The sky is blue, even if the air is perfectly clean, because light is scattered randomly, depending on the color. The effect is called 'Rayleigh scattering', and it states that a light ray can travel unimpeded through air for a certain distance, and then it undergoes scattering, which means the light ray randomly changes directions, without changing color. Just how far depends on the color: blue light has a short scattering length, and red light has a much longer scattering length. That's why the sky is blue: all that blue gets rattled around and comes at us from all directions, and the effect also makes sunsets red, when the light has to travel through much more air. So your first choice would be a red laser. Next is air quality. When choosing a color, you want to avoid wavelengths that are strongly absorbed by all the other things in the air: water vapor, CO2, molecules of smog, dust etc. Some of these molecules can vibrate at particular frequencies (=colors), and you want to look those up and avoid them.
Next, it matters *how* you are looking at the distant laser: suppose you arein a noisy crowd, and then a high-tone bell rings: you can hear the bell because none of the voices in the crowd produce such a high frequency - the little bell really stands out. If you made ear protectors that block out all low-frequency sounds, it would be completely quiet until the little bell rings. Similarly, if you know exactly the frequency (or wavelength, or color) of the laser you're looking for, and you get a filter that blocks all colors except for the one you want to see, you have a much better chance of seeing your laser by.
Finally, if all else fails, get a more poweful laser. In the real world, lasers can go tens or hundreds of miles through air. For example, lasers can be used to communicate from the ground to satellites, hundreds of miles up.
Q: When a star dies for how much longer will we still see it shining?A: It depends how far it is! The farther it is, the loger we will see it https://www.youtube.com/watch?v=ELD_JvSk6xU#action=share
Q: Why did the inventors of the blue LED recieve a Nobel Prize?A: " Although red and green LEDs had been around for many years, blue LEDs were a long-standing challenge for scientists in both academia and industry. Without them, the three colours could not be mixed to produce the white light we now see in LED-based computer and TV screens. Furthermore, the high-energy blue light could be used to excite phosphorus and directly produce white light - the basis of the next generation of light bulb.""What's fascinating is that a lot of big companies really tried to do this and they failed," he said. "But these guys persisted and they tried and tried again - and eventually they actually succeeded."https://www.bbc.com/news/science-environment-29518521
Q: Why is there multiple rainbows when I look through the glassA: Not clear what the situation is here. Was this when you looked through the spectroscope (the plastic tube with the slit on one end and the piece of CD on the other? Could you rephrase your question? (Hubert)
Q: Why is it called the electromagnetic spectrum? What does light have to do with electricity and magnetism? Can different light sources interfere with each other and change their wavelength?A: The answer to the first two questions are given elsewhere on 'Ask Anna'. Here is the answer to the third question: yes, if you mix two light beams with slightly different frequencies, you can make a new frequency equal to the difference between the two incoming beams, but it is not easy. The reason is that the frequencies are so high (Here is a video where two beams get mixed where the difference between frequency is just 1 part in 6x10^15. (Hubeert)
https://www.youtube.com/watch?v=CuGEPkI117s
This video is a link from this discussion: https://physics.stackexchange.com/questions/56144/can-light-waves-cause-beats
Q: Do gases other than oxygen emit a light?A: Yes, you can make any atom emit light, gas or not. For example, the orange highway lights have bulbs with Sodium vapor, which emits strongly in the orange. The older bright white street lights have bulbs that work with Mercury vapor. Neon lights of course use Neon gas. Some strobe lights use Xenon gas. Helium gas is special because it was first discovered in Sunlight, because of its unique spectrum of light. Only after that was Helium found on Earth. If you want to make a light with Iron, you can do that, but first you have to heat it up to where it melts and then vaporizes - not easy, but it can be done. The Mars Rover, for example, has a powerful laser on board with which it can vaporize rocks, and then a camera looks at the light coming from the vaporized rock, and by the spectral colors it can tell which atoms were vaporized.
Q: I remember using the convex lenses today, and how the view from outside our windows were flipped vertically on our index cards. I also remember hearing that our eyes see the world upside down, but our brain is what allows us to see things right side up. Is the shape of our eyes similar to the shape of the convex lenses?A: Yes, your eye uses convex lenses to make the upside-down image on your retina. The first one is right in front: if you close your eyes, and place your fingers on your eyelid, and then move your eyes left-right-left-right, you can feel that the front of your eye has a convex surface right in the center, about 1/2" diameter. This one is called the cornea, and it cannot change shape. The second one is behind the iris and pupil. This one can change its shape, becoming flatter or more rounded, this change happens when you change focus between looking at something in the distance (flatter), and something close up (rounder). The cornea plus the lens produce the sharp image, and allow you to focus on close-by objects or distant things (but not at the same time). Q: How do we know that light is electromagnetic radiation?A: It's easy to tell that radio waves or microwaves are electromagnetic, because we can see the batteries, power cord, copper wires and all that. For light, this is not so clear. But we know that atoms are made of electrically charged object (the protons in the nucleus and the electrons 'in orbit'), and if we hit them with a powerful electrical punch, light comes out. The inverse can also be done: hit material with the right kind of light, and electricity comes out. These things tell us that light and electromagnetism are part of the same phenomenon.
Q: I was just wondering about the stipend? A: The stipend for the 2020 Summer Phsyics Camp will be $300 when Materials are returned. If student decides to keep the cool Raspberry Pi and arduino, then stipend will be witheld.
Q: How long after a meteor hits will there be shock waves for an average size?A: The shock waves that travel through the earth (P-waves) travel 5-8 km/second, so if you are at 100 miles distant, the shock wave would reach you in about 25 seconds. The schock wave in the air travels at about 1/3 km/second, and would arrive about 8 minutes later. (Hubert)
Q: Can radioactive isotopes be used to treat other things besides cancer?A: Cancer is the primary target for using radioisotopes in medicine. Radioisotopes are also explored for treating drug resistant bacteria and some viruses including HIV. Radioisotopes are also used to sterilize equipment because it doesn’t leave any residue.A: Yes: radioactive iodine-131 is used to treat hyperthyroidism, when the thyroid produced too much thyroid hormone. But besides treatment, radioactive isotopes are used for diagnosing diseases. For example, Carbon-14 can be used to diagnose stomach ulcers caused by the bacterium h. pylori. Radioactive isotopes are often used to find tumors, before treating them. (Hubert)
Q: How long does the radiation in each materials last? Also how intense over a period of time?A: The half-life, or the time for 50% of the radioisotope to decay, is usually relatively short for medical isotopes, since the goal is to have the decay occur on a similar timescale to the biological processes used for targeting. For example, actinium-225 has a half-life of 10 days, which is well matched to the time for the peptide targeting to occur from the blood stream. Rubidium-82, which we also make (via the parent strontium-82) has a half-life of 75 seconds. This is used for cardiac imaging, and is just carried in the blood stream and gives an image of the blood flow and any blockages as the blood moves through.A: This depends on the isotope. Each radioactive isotope has a *half-life*. This is the time it takes for half of the atoms to decay. Halflives range from fractions of a second to illions of years and longer. In medicine, you don't want to choose one with a short halflife, because then you don't even have time to get it from the production lab to the hospital. Also, you would not want to have one with a long halflife, since then your poor patient would be radioactive for months or longer. So the intensity depends on the halflife. If the halflife is one day, then the intensity of the radioactivity is cut in half after one day, and after 10 days, only 1/1000th of the activity is left. (Hubert)
Q: Are sensors installed in the manipulators to help with maneuvering objects?A: No sensors, you just have to practice! The thickness of the windows makes this quite challenging. For the chemists, they practice pouring with water to be sure they have the depth perception correct. We also have new people remotely clean the hot cells, bag up waste, or set up equipment to get their skills refined….tasks that need to be done but where it is OK to make a mistake or take your time. Working with a batch or product to go out can be rather stressful.
Q: What do you concentrate the isotopes with?A: During chemistry, we use pretty common techniques to concentrate materials. We may gently boil a solution in the hot cell to evaporate off some of the solvent (water or buffer) and reduce the volume, which concentrates the isotopes left in the solution. Most of our purification techniques rely on column chromatography, where the a solution is poured through a set of tiny beads with a ligand on the surface. As the solution passes over the beads, the ions in solution interact with the surface of the beads. Some will stick (what you want) and others will pass through (what you don’t want). This can be used to essentially concentrate the dissolve ions onto the column. If you change the solution, you change the affinity of the ions…or you can then pull off your desired isotope into a smaller amount of solution.
Q: Where were other parts of Cassini made from, like other labs or countries?A: Cassini/Huygens was a collaboration of teams from NASA, ESA (European Space Agency), Italy, Austria, Germany, Spain, France, the United Kingdom, Norway, Finland and the United States.[Hubert]The other instruments were made at institutions from all over the US and even Europe. Some examples: The cameras that took the amazing pictures were made at the NASA Jet Propulsion Laboratory (JPL) in Pasadena, CA. JPL is also the NASA center which oversaw the whole mission. The magnetometer, a device used to measure the magnetic fields through which Cassini flew, was made in the UK at Mullard Space Science Laboratory (MSSL). Mullard also made the instrument that detected electrons flying around in Saturn's magnetosphere. The instruments designed to detect high-energy particles were made at Johns Hopkins University's Applied Physics Laboratory in Maryland. And the radio antennae that detected lighting on Saturn were made at the University of Iowa. Cassini, like most of NASA's space missions, was actually made up of instruments from all over the world. So you don't actually have to be working at NASA to be a "NASA scientist". [Dan]
Q: What are some other camps or programs that are recommended so that we can further delve into the world of science?A: Every area and school has different opportunities, but I would suggest FIRST robotics as a way to learn more about robotics and I would check to see if there's any companies or government research organizations (like NASA) that take high school interns - some do! [Space Gal]

Q: Are women less likely to be considered for shows that have to do with science?A: It depends. If you're working to be the host of a science show on a major network, yes. It is more difficult because the vast majority of their viewers are male and the Network execs assume that those viewers will only respond well to hosts who look like them. I would argue that bringing in more female hosts would allow them to capture an audience they haven't reached previously. I'm hoping that this changes little by little each year. [Space Gal]
Q: What does a physicist do and how can I start looking into a career?A: The range of things physicists do is vast. But you could divide it into three groups: working in industry, working in research, and working in academia. There is overlap between these categories. Industrial physicists do research to improve processes, or to invent new ones. Many physicists at universities do both reaseach and teaching, and many searchers (for example at the Lab) mentor students and postdocs. Here is a link to a booklet from the American Physical Society about careers in physics: https://www.aps.org/careers/guidance/guide/index.cfm And you can always contact me. (Hubert, a physicist)
Q: Where do Saturn's Rings come from?A: Now there's a great question, and the answer is: we don't know. Look at this article: https://www.universetoday.com/107353/where-did-saturns-rings-come-from/ And I always tell my students "Ten or fifteen years from now, when you are a young scientist and maybe the world's expert on the rings of Saturn, you should come back and tell me the answer". (Hubert)A great question! Saturn's rings are made of water ice particles ranging in size from fractions of a millimeter to a few meters in size. The rings are only about 100 m thick! That's about the length of a football field. Nobody knows for sure how the rings formed. Two leading theories are 1) a largish moon got too close to Saturn and got ripped apart by Saturn's gravity into fragments, or 2) The rings are leftovers from Saturn's original formation 5 billion years ago. [Dan]
Q: When we talked about galaxies, we saw that their blue colour indicated stars were forming and red colour indicated passive galaxies, or essentially the death of stars, is that correct?A: Almost! ‘Blue’ galaxies are blue in the visible spectrum because the galactic light is dominated by young, hot stars whose spectrum peaks in blue. The ‘red’ ones are red not because stars ‘die’ or explode. The galactic spectrum is essentially only composed of stellar light of old stars – they are much cooler than the young ones and their spectrum peaks in the red. The red galaxies are also devoid of gas which can fuel future star formation, hence there is negligible chance for them to get the blue light in the future. [Asia]An example of a blue galaxy: M51 https://www.nasa.gov/sites/default/files/thumbnails/image/m51-and-companion_0.jpgAn example of red galaxy: M87 https://upload.wikimedia.org/wikipedia/commons/0/07/Messier_87_Hubble_WikiSky.jpg Q: How does the color of a galaxy tie into a redshift and blueshift when objects are moving either towards or away from you? I'm not sure whether these two concepts are even connected, but in that case why are the colors blue and red so often seen?A: The colour of a galaxy is in part tied to its distance away from us. Practically all galaxies in our Universe move away (recede) from us in a cosmological sense. The further away in distance they are, the more quickly they are receding (the famous Hubble’s Law, discovered by Edwin Hubble and demonstrated in his plot: https://www.pnas.org/content/pnas/101/1/8/F1.large.jpg). This means, that for far-enough galaxies the reddening of spectra from the expansion of the Universe is relevant, making a star-forming galaxy seem red overall. However for galaxies that are closer to us, like the ones we saw images of during the talk (redshift of 0.08) the observed face-value optical colour of the galaxy is determined by its composition (i.e. whether or not there is active star formation, stellar explosions, ionised gas, old and young stars).When we refer to a galactic colour in astrophysics, we also mean the ‘rest-frame’ colour of it, not the face-value observed one. This means that in the description of colour we already account for the reddening due to redshift. In the scientific literature, the colour is defined as a ratio of fluxes in two sections of a spectrum, hence describing a relative contribution of, say, red with respect to blue or green. The reason why the colours red and blue are seen so often is because galaxies tend to spend very long times in each evolutionary state, hence statistically there is a higher chance of finding a galaxy which is either blue or red than for e.g. in-between. When we look across cosmic epochs, the distribution of galaxy colour changes from blue-dominated (i.e. more blue objects than red) to red-dominated and for this reason we interpret the blue-to-red transition as an evolutionary sequence. Also, the definition of something being ‘blue’ and ‘red’ spans a broad range of actual ‘colour values’, so, if you will, our definitions are rather ‘loose’. [Asia] Q: What are the "green" galaxies marked as "green" for? If blue means birth of stars and red no more star formation, do the "green" names just indicate neither of the two?A: Good job, you nailed it. The name ‘green’ comes from a definition drawn in a colour-magnitude diagram. Actually, a good blog entry answering this question and the one above is here: https://blog.galaxyzoo.org/2014/02/21/the-green-valley-is-a-red-herring/. Everything all the way up to the ‘quenching’ section is spot on, however newest research puts a couple of small question marks on the final part of the blog. [Asia] Q: Because of the redshift, we know the universe is expanding, right? What gives us a redshift indicating the universe is expanding?Yes, the redshift tells us that objects are receding (moving away) from us. This is, however, in a cosmological sense. It means that the object is not moving away from us through space, but rather *the distance between the object and us is increasing* as the photon travels, due to the expansion of the Universe. One way of visualising it for yourself is taking a balloon, blowing it up to a small ball and drawing two dots A & B with a marker. Then draw a waveform representing a photon (a wave with, say, 3-4 crests and throughs). Then blow up the balloon a bit more and you will see how A and B get farther away from each other, although their position on the balloon surface does not change. These are your galaxy and yourself affected by the expansion of the balloon, 2D Universe. Now look at the photon – it got stretched, the wavelength increased but not because the wave crests were arriving at increased separations in time due to the motion of the far-away galaxy (like in the case of Doppler Effect). The wavelength increased because the 2D spacetime of the balloon Universe got stretched. Now, our Universe is a 3+1 dimensional version of the balloon, so it’s harder or near impossible to visualise how redshift works in our scenario. A neat explanation of what we are discussing is here: https://astronomy.swin.edu.au/cosmos/c/cosmological+redshiftSo now that we know what the cosmological redshift is, let me address how we found the expansion of the Universe. Thanks to multiple people like Edwin Hubble we were able to measure how far an object is from us and measure the redshift of emission lines, finding that the further away an object is, the more quickly it is receding from us. An indication that we are, indeed, in the balloon scenario. [Asia] Q: Where do we find a redshift?We find the redshift by comparing the wavelength of emission/absorption lines observed for an object and their ‘rest-frame’, laboratory wavelengths. Each atom has a unique emission/absorption line pattern dictated by quantum mechanics, which we can calculate theoretically and measure in a laboratory. This pattern comprises of several lines, which is a template of lines spaced at particular wavelength intervals. We measure a spectrum of a distant object and overlay such template, shifting it around the spectrum until it matches the observation. Once we have found the match, we measure the difference between the laboratory wavelength values and the observed wavelength values to determine the redshift. [Asia] Q: I'm wondering how exactly the electrons jumping levels relate to different colors of emitted light. Does the number of levels jumped down relate to wavelength and therefore relate to color emitted?Well done! Although, a couple of small clarifications. Firstly, the energy levels are not evenly spaced (so we do not count the ‘number of levels jumped down’). Secondly, the probability of an electron making a shift between two particular energy levels is determined by quantum mechanics. There are also some transitions dubbed as ‘forbidden’, i.e. the probability of an electron making a shift is very small. For this reason, electrons hang out in the higher energy state for a longer while (which we referred to as a meta-stable state in our discussion), before they finally fall down – that’s the reason why your phosphorescent paper was showing a glowing pattern which didn’t vanish immediately. [Asia]

Friday June 12

Q: Is the ocean salty like the one on Earth? Is it drinkable?I really like this question! I assume you're asking about the ocean beneath the ice crust of Enceladus? The ocean is probably not as salty as Earth's oceans, because our oceans are salty due to salts in eroded soil and rock carried by rivers into the oceans. The rivers themselves are formed by rain. Since there is no rain or significant erosion going on underneath the ice on Enceladus, the water there is probably more like freshwater on Earth. As to whether it would be drinkable by humans, that is an open question. Even if it isn't salty, it could be fairly acidic, since there are thermal vents on the ocean floor of Enceladus bubbling hydrogen gas up through the water, which has the tendency to make the water acidic. We don't know if it is too acidic to drink, though. After all, Coca Cola is quite acidic (it will strip the enamel off your teeth if you drink too much!), but we can certainly drink that. [Dan]
Q: How did Heather Bottom get involved with Dragon Space mission and Space X? A: While I was in graduate school for Aerospace Engineering at Caltech (at the time, I had obtained a master's degree, passed qualification exams, and was working torwards my PhD), I started looking into full time work. I wanted to put my engineering skills to use in an applied environment (i.e. at an engineering company). And at the time, I wasn't interested in getting a PhD. Engineers can, many times, work in the industry without getting a PhD. I submitted my resume to some companies and got the job with SpaceX. The company originally wanted to hire me as a thermal engineer (given my experience in grad school), but in the end the System's Engineering job was a better fit for my interests and skills. I took a small risk because I also didn't fully know or understand what a Systems Engineer's job entailsd. The job was hard to turn down, especially given the mission goal to send US astronauts to the ISS. And from there, I worked for about 3 years with the Dragon Capsule and astronauts who are now at the ISS. [Heather Bottom]
Q: What do you recommend doing before you graduate that helped you in college or helped you prepare for college?A: I think one of the best things you can do to be ready for college is to be open to new ideas and new experiences. Getting out of your comfort zone can be very challenging, but this can open up future opportunities that you can’t even imagine today. Meet different people, ask questions, and don’t be afraid to try something different. [Eva]Ask for help when you need it! Building up a good support system and looking for people who can guide you will both build connections and help you to succeed. [Ellen]Absolutely, great addition! Everyone needs help sometimes. And then don’t forget to help others when you are in a position to do so. Together, we accomplish so much more. {Eva]
One thing to do to prepare for college would be to think seriously about the types of things you enjoy doing and the types of classes or subjects that interest you the most. Dive in deep and read all you can about those areas; learn all you can! Do all you can! There are so many resources out there today. Take on new projects that you enjoy doing... new, old, different. College will be busy and you won't have as much free time to enjoy slowly learning and discovering new things on your own time, so use the time now to do so. The more you learn and explore now, the more prepared you'll be. Even if the knowledge isnt directly applicable to the work in college, the experience will shape you in ways you cannot imagine. You'll bring something to the table that other students may not have thought about already. [Heather Bottom]
Q: Would someone with less tumors still benefit or choose the medication that is currently being made with radiation, or would it be smarter to opt for surgical removal just because the cancer hasn't spread throughout the body as much? Is the Radiation too much for the brain to handel, as in this treatment is not an option for brain tumors?A: The Ac-225 is in very early clinical trial, so the use is very limited right now to patients who have exhausted other treatment options. Surgical removal of a large mass tumor may be a better option for some patients; alpha emitters are not usually a good choice for larger tumors because that type of radiation doesn’t travel very far. Treatment of brain tumors is further complicated because many drugs (whether radioactive-based or more traditional) do not pass the blood-brain barrier, and so getting medication to the tumor site is very hard. And as you suspect, the brain is very sensitive to side effects. Selecting a treatment regimen is very individualized and I am certainly not the expert! We try to engage with people who know more about treatment so we can bring new tools to the table as options. [Eva]
Q: How much weight can tension hold before breaking (with the basic mesh we used today?
Q: What exactly is an arduino? A: Matt will have another slide about this in Mondays presentation. Hopefully it will clear things up. An "Arduino" is actually a specific brand of microcontroller board. They were the first (and probably still the best) so people often refer to any generic board as an "Arduino". Your board that says ELEGOO UNO R3 is what we are referring to when we say "arduino" for this class. The Elegoo is actually a cheaper knock off of an official Arduino brand board but it interfaces with the software and sensor, motors, etc. just the same. [Matt]
Q: My internet cut out when Dennis was explaining turbulence, so what is it, and what does it do?
Q: If there are just 2 digits of bits, how do we know what they are supposed to represent?A: [Joan] Good question! Yes, everything, everything, everything inside the computer is either 0 or 1 (or on/off, or true/false). A transistor is either holding an electric charge, or it is not. An electrical current is either running through the wire, or it is not.So, all the ordinary things we do in life (like writing words) needs to be represented using only zeroes and ones. Sixty years ago, people got together and agreed to use the ASCII coding scheme, so that we would all understand each other. In ASCII,the letter ‘A’ is represented by the 8 bits: 01000001and the letter ‘B’ is represented by the 8 bits: 01000010and the letter ‘a’ is represented by the 8 bits: 01100001etc., etc. for all the symbols on your keyboard.There is no particular rhyme-or-reason to why we give a particular code to a particular symbol. Any coding will work, as long as we all agree on it, so we can all understand each other. (Actually, there are some reasons, but they are too low-level for us to think about here)You can google “ASCII table” to see all the ASCII codesBecause ASCII uses 8-bits per symbol, ASCII can only represent 2^8 = 2**8 = 256 symbols.More recently people have invented Unicode which uses 16 bits per symbol. This allows 2^16 = 65,536 possible symbols. Wow! That is a lot. But there are so many different languages in the world (Chinese, Arabic, Greek, etc.) that don’t use A, B, C, so we needed more symbols thanASCII can give us. OK, that finishes PART 1 of the explanation! Let us move on to PART 2 Computers do many amazing things, but they originally were used as number-crunchers (e.g., computing the vibration of a bridge during an earthquake. Will the bridge fall down?) So, we want to represent numbers inside a computing, and we only have the two symbols 0 and 1. We could use ASCII, but there is a better way.In ordinary life we write down numbers using base-10 (or decimal) notation. For example, consider the following numbers:10 which is ten100 which is one hundred1000 which is one thousand.We could also write these as:0010 which is ten0100 which is one hundred1000 which is one thousand.By putting some leading zeros in front. The zeros in front do not change the value. We usually don’t do this, but we can. Our number system is positional. The meaning of the vertical line (i.e., the symbol 1) depends on where it appears (i.e., in which position it appears).0010 is ten, because the symbol ‘1’ appears in the second-column-from-the-right-end.0100 is one hundred, because the symbol ‘1’ appears in the third-column-from-the-right-end.We talk about the positions as the “one’s place”, the “ten’s place”, the “hundred’s place”, etc.0362 is three-hundred and sixty-two0623 is six hundred and twenty-three.The meaning of the curly symbol ‘6’ (is it sixty, or six-hundred?) depends on where the ‘6’ appears. This positional notation is different from Roman numerals. In Roman numeralsCCXXXVII is two-hundred and thirty-sevenXXIIII is twenty-fourThe meaning of the symbol ‘X’ is ten no matter where it appears in the written number.So now that we understand our positional number system (decimal, using base 10), we can do exactly the same thing in binary (i.e., base 2).In decimal we have a “one’s place”, a “ten’s place”, a “hundred’s place”, a “thousand’s place”, etc. These are the powers of 10: 1 = 10^0, 10 = 10^1, 100 = 10^2, 1000 = 10^3, etc.In binary we have a “one’s place”, a “two’s place”, a “four’s place”, a “eight’s place”, etc. These are the powers of 2: 1 = 2^0, 2 = 2^1, 4 = 2^2, 8 = 2^3, etc.So the binary number 1101 is:One copy of 1000 (ie the value 8)One copy of 0100 (ie the value 4)No twoOne copy of 0001 (ie, the value 1)So the value of the binary number 1101 is equal to 8 + 4 + 1 = 13Any number written in normal decimal (base 10) notation can be converted into binary, and vice-versa. If you pull up the Calculator on your computer, you probably get a very simple calculator, with add/sub/multiply/divide. Click on the top left corner and it will let you choose other kinds of calculators. A Scientific Calculator will have buttons for sine and cosine, and logarithms and square roots. A Programmer Calculator will have buttons that let you move between decimal, binary and hexadecimal (base 16) notationQ: If the jar were to have no bottom, would the water come out of the mesh if tilted upside down?

Monday June 15

Q: John Sarrao, what school would you recommend, UNM or NM Tech? Which one would be better to have a degree in and work at LANL?A: Both UNM and NM Tech are good schools. LANL has many interactions and collaborations with both schools, and many graduates from Tech and UNM work at the Lab. I don’t think you can go wrong with either choice. They both have strong science and engineering departments. UNM is larger than NM Tech, as is Albuquerque compared to Socorro. So, if you’re sure about what you want to study and prefer a smaller campus with fewer students per faculty, NM Tech could be the answer. On the other hand, if you prefer a larger school with more options, then maybe it’s UNM. If possible, I’d recommend visiting both campuses, meeting some students who are studying what you think will be your major as well as some faculty, and seeing what feels right for you. [John Sarrao]
Q: Hannah Mohr, How long does it take to take a picture in space? Does it require a lot of money?A: t took several years of research to design the satellites and launch them into space before we could get a picture, but now that we have them in orbit we can get a picture in about a day. It costs quite a bit to build, launch, and operate your own satellite, but there are many universities these days that have programs where undergraduates can be a part of that process! I hope this helps! It's always great to hear when people are excited about space research.
Q: How many arches are there in Arches national park?Q: There are more than 2000 arches in Arches National Park. When Arches first became a national park in 1971, there were only about 90 known arches. It then took 20 years of searching from park rangers and visitors to find, document, and name all of the arches hiding within the park. Since the last complete surveys in the early 1990s, many arches have likely collapsed or been created, so no one knows the exact number of arches today. Fun fact: an opening in the rock must be at least three feet long to be officially counted as an arch!
Q: How do we change the circuit in order to have multiple LEDs connected such that they also blink? If we did this would they all blink at the same time or would they all be slightly delayed? Great question! Read my answer to the following question. Lesson 4 in the manual is essentially hooking up 3 LEDs to the Arduino. You can code to have them all come on at once, or vary which light is on and also how bright it is. Another answer to your question is that you can hook up multiple LEDs. The negative end goes into the blue strip, the positive LED end and one end to a resistor go into the same column. Then the other end of the resistor goes into a different port. You then add code to define the LED port (we used int ledPin=13, for the first one) and add code to set that port HIGH and LOW. [Matt]
Q: What are some other activities we can do with the arduino?Take a look at the manual that is one the CD that is in your kit, I also have the manual posted to http://chamisaelementary.com/robotics/Documents/Super%20Starter%20Kit%20for%20UNO.pdf . It is full of different projects for you to do. We did Lesson 3 on Monday. Lesson 4 is a good extension of our previous lesson and also teaches another important concept about "pulse width modulation". It is essentially a similar circuit but you hook up 3 LEDs and give them variable power rather than full on or off. The code is also on the chamisaelementary web site if you get stuck with that. But feel free to jump around the manual and choose whatever project you would like to do. All of the code is also on the CD, and about half of it is on the web site. [Matt]
Q: What are some materials that can be used for 3D printing and could we possibly have 3D printers at home one day? Is anyone working towards that?A: That’s a great question. The most common 3D printing filament for home use is called PLA (Polylactic Acid); it is a corn based biodegradable plastic that is capable of making highly detailed parts. One issue with PLA is that it is not very tough (yes that’s an actual engineering term, think it will not survive being hit with a hammer or dropped too hard) and it will warp if left in a hot location (car in NM summer). When home makers need tougher or more heat resistant prints they typically turn to PETG (disposable water bottles are often made of this) or ABS (plastic that Legos are made from). There are positives and negatives to each of these materials as well. 3D printers at home are becoming more and more common. Very few people have the exact printers that I showed in the video at home because they all cost between $5,000-$50,000. Currently home printers range between ~$200 and ~$5000 which is still a lot of money, but it is more accessible than it used to be. [Graham]
Q: Why do you think that Geomorphology or Geology is one of the scientific fields that lack diversity and is staying behind? A: people aren't completely sure. While the number of geoscience PhDs earned by women has gone up greatly- in many subfields it's about 50% women, 50% men earning PhDs now- the geosciences are the least ethnically and racially diverse of all the STEM fields. Some people think that one of the reasons geology lacks diversity compared to other STEM fields is because there's a perception that geologists have to be outside all the time and do lots of fieldwork. While this used to be the case, maybe 100 years ago, modern geology includes much more math, computer and experimental work in addition to fieldwork. This misconception also hasn't been helped by the "classic geologists" (most of whom are white males) who perpetuate that stereotype in geology departments across the country, or by geology professors who don't take into account students' diverse experience levels with the outdoors and lead them on unpleasant or dangerous field trips. People have recently started looking into the economic and social barriers that prevent minorities from spending time outdoors. Hopefully we can both improve access to the outdoors for everyone, and help people better understand that doing geoscience isn't all about eating dirt and going on long, treacherous hikes!
Q: What are some ways in which we can get into coding and computing? Who was the first female computer scientist?A: The idea of a computer traces back to about 1840. Back then there were lots of machines (steam engines, looms for weaving) but a computer is fundamentally different from these because it is a programmable machine. Instead of doing just one task, like weaving cloth, it can do any task that you program it to do. Charles Babbage of Great Britain was the first to try to build such a machine. His friend and colleague, Ada Lovelace, is called the “first programmer” because her letters contain actual programs for this machine. Babbage was never able to actually build his machine, because the technology of that day was so much more primitive than now. For example, there was no electricity. Ada Lovelace was remarkable not only in writing programs, but in realizing that the computer could process any information. Babbage was thinking of the machine purely in terms of doing arithmetic (add, sub, mult). But Ada said that the machine could process and transform music. About 1960 a computer programming language called ADA was developed. But it never was widely used.The modern computing age started with the construction of the ENIAC in 1943-1946. Because this was war time, many of the programmers for ENIAC were women. One particularly famous one was Grace Hopper. I guess you could call her the first woman computer scientist. She made a fantastic contribution to computing. She invented the compiler. Earlier we wrote some Python source code (i.e., Projectiles Activity). All source code needs to be translated into zeros and ones in order for the computer to understand and execute the code. This translation used to be done by people, by hand. And extremely difficult it was too! Grace Hopper invented a program, called a compiler, that will automatically do this translation. Think about it….a compiler is a program, that reads in a program (perhaps a Python program) and outputs a program (the zeros and ones of the machine code). A program manipulates a program! This was truly a revolutionary idea. The project Grace Hopper worked on was funded by the Navy. She had a long career in the Navy, and a battleship was named after her in her honor!
Q: How do arches form:That's a great question and one that isn't so simple to answer. [Paul]
I like to think about arch formation as a three-step process:Start with lots of large sandstone cliffs that soak up water every time it rains - even in the desert, water is the key here!Water flows through the rock and dissolves the cement holding the grains of sand together.The wind blows away the loose uncemented grains of sand and gravity pulls down bigger blocks of rock to create a circular opening and an arch!More great information can be found from Arches National Park if you want to learn more:Why are there so many arches at Arches?Types of arches

Tuesday June 16

Q: How do put a carbon filament into a build? Like do you have to load the printer a different way?A: The printer has two nozzles, one nozzle runs filament just like in the other printer I showed; except that the filament in this printer is a nylon plastic that had chopped carbon fiber added to it at the factory. Even with this carbon fiber, the printer works in the same way. The second nozzle has a spool of carbon fiber running through it. The fiber is about the same size a fishing line. The printer melts the string of carbon fiber into the layer below it and has a blade to cut the fiber at the end of the line. [Graham] Q: Can you make metal objects using the 3D printer?A: Yes! We can make metal parts using 3D printing. The most common method of 3D printing with metal is called laser sintering or laser melting. The way it works is that the printer lays down a very thin layer of powdered metal a very powerful laser then traces over that metal to either weld or melt the small metal particles in the powder. The printer then spreads another thin layer of powder and the process repeats. This short video gives a pretty good overview of how laser sintering works (https://www.youtube.com/watch?v=yiUUZxp7bLQ&t=76s). Another method of 3D printing with metal is similar to FDM printing which is the type of 3D printing I showed in my video. In this type of printing, metal powder is combined with a wax/plastic binder and turned into filament. This filament is 3D printed using the same concepts as normal plastic printing. After the part is finished printing, it is placed in a solvent bath which removes most of the binding material from the print. The part is removed from the solvent bath and placed into a sintering furnace. The furnace heats the part up to ~1200 oC which burns off the rest of the binder and welds the small metal particles together.
Q: Is there a way people could program computers to learn how to teach other computers by training them?A: It is something that the AI community is interested in! We work on things like using machine learning methods to help train other machine learning methods, so it is an active area of research.[Katie]
Q: When training robots to do certain things at your job, are there different people doing different steps? (ex. one that builds the robot, one that programs sensors, and someone who trains the robot)A: Yes, we're often working together in a team, where different people have different roles on the projects. In the case of the robot, there were several people who worked on different aspects of the robot, including building the robot and sensory [Katie]
Q: What special classes in college should I take if I were to work with viruses like COVID-19?A: I would say virology, immunology, molecular biology and biochemistry. [Karissa]
-Q: So would a virus problem, have rapid growth or rapid decreases, like exponentially? Would it look really pronounced and what conditions would you have to take into consideration when working on a problem like that?A: Yes, corona virus did have an exponential increase when it started in each outbreak area. This depends on how contagious the virus is and how easily it spreads. There is a number called R0. If one person has coronavirus, R0 is the average number of people that will catch corona virus from that one person. When R0> 1, then it tends to grow exponentially. When R0<1 (for example R0=0.5), it tends to die out. In some countries, it appears to have decreased exponentially . You can look and the growth and decay rates on the John Hopkins website: [Karissa]https://coronavirus.jhu.edu/map.html
-Q: Can DNA be used to predict and understand people's personalities and their hobbies?A: Yes, in principle DNA could be used to predict and understand people's personalities and their hobbies - especially if you understand the epigenetic marks on the DNA and you have data inside the brain in real time. However, the current technology and scientific understanding is not yet advanced enough to do this. Maybe you could be the first one to predict a personality! {Karissa]
Q: What kinds of things does a software engineer do?A: A software engineer can also be called a programmer. It depends on the industry I think. In video games I was titled "programmer", in academia and the electronics industry I've was titled "software engineer". In any case, those two job titles put together make a good description; an engineer that programs. Ultimately, I solve problems by programming. The girls were doing that in the Arduino class before my presentation. One of the reason I went into programming is the diverse nature of the trade. Just about every technical industry involves programming in some aspect. As a programmer you can work on everything from spaceships and science to movies and video games. [Hawi]
A: Software engineers explore the structure and behavior of computer programs and computer systems.The Institute of Electrical and Electronics Engineers (IEEE), world's largest professional organization includes a Computer Society (a sub-organization) that publishes the Software engineering Body of Knowledge (SWEBOK). The SWEBOK includes the following subjects, any and all of which Software Engineers should be familiar with:
Software RequirementsSoftware DesignSoftware ConstructionSoftware TestingSoftware MaintenanceSoftware Configuration ManagementSoftware Engineering ManagementSoftware Engineering ProcessSoftware Engineering Models and MethodsSoftware QualitySoftware Engineering Professional PracticeSoftware Engineering EconomicsComputing FoundationsMathematical FoundationsEngineering Foundations
Many of these areas involve creating and bringing to life abstract models of the subject area or writing code in programming languages and programming scripts.
Hope that helps... {Terri]
You can probably access this resource....https://ieeexplore.ieee.org/xpl/RecentIssue.jsp?punumber=32-Q: Why is that different functions of the soundboard such as playing music or turning on the LED lights take different amounts of energy?A:The circuit on the little board that plays music is made up of many electrical components, such as transistors, resistors, capacitors, memory cells, and finally a small speaker. The part that makes the LED blink anso uses many components like that.. None of these are 100% efficient, and they all lose a little bit of energy when they are working. How much energy is lost also depends on how the circuit is designed. Some designs are more efficient than others.-Hubert
Q: What are some benefits and drawbacks of going to a local school, such as UNM or NM Tech. Ashlee: I went to a local school (I'm a NC native, so it wasn't one of the NM colleges). If the school is accredited (for example, for engineering, you want to make sure it is an ABET accredited program), then there are no major drawbacks to going to a local school. One thing I liked for going to a local school was proximity to my family. I was far enough to live on my own, but close enough to visit for a weekend if I wanted to. I currently am in grad school at an out-of-state private university and being further away does make it harder to visit. If you attend an in-state public school, the tuition is often cheaper than out-of-state schools or private schools. I think also the culture at a local school will be more familiar. This can be a pro and con, depending on what you are looking for.I have also attended a community college (2-year college, Associate degree), which has the potential for transfering to a 4-year (Bachelor degree) college. This is another local option, which I highly recommend. Community college tuition is often cheaper and the class sizes are smaller, so it becomes more personalized. Downside is that they will not grant a 4-year degree, but that is not necessary for all types of jobs and the credits can often transfer.
Tiffany: I actually choose to come from out of state to UNM because it was affordable (more affordable than staying with my own in-state options). At the end of the day, hard work, a good resume and good recommendations are more important than where you go to school. You do not have to go to a fancy (and typically very expensive) private school to get a good job, especially in science. One of the great things about UNM is that it is near Sandia National Labs, the Air Force Research Lab and Space Weather group and Los Alamos National Labs, and has active collaborations with all three. Being near these places makes it easier to get internships, and making friends with professors and advisors can help you get opportunities into those places.
Isabella: I did not go to a local school in New Mexico, but I think I can provide some perspectives on it. If you grew up in New Mexico, I highly encourage you to branch out and experience college in a different state (if possible). College is usually only ~4 years, so it is a great opportunity to diversify your experiences and possibly see what other cultures are like. There may be some drawbacks though, such as being farther from family and having to pay more if you are out-of-state. Some schools offer scholarships though that can make the price the same as if you were in-state (for me, this was usually specific to the school, and depending on the situation, they may offer it to you without you needing to apply separately!). When I was applying to college, I received a scholarship from my local rotary club. Ask your high school academic counselor of any local opportunities for scholarships! College is a huge period of growth and offers new/exciting challenges; take advantage of this time and make the most of it. :)
I hope this all helps. Feel free to give out my email if the students would like to chat more.
Patrick: I found going to New Mexico State University beneficial. Classes were sort of small, so student/professor ration was good.Some of my high school classmates also attended NMSU which made study groups a bit more familiar.The lottery scholarship is now a benefit to cover tuition…assuming criteria are met and sustained.I cannot cite a drawback. I would likely make the same decision today as I did decades ago.
Cassandra: There are financial benefits to studying locally because the tuition may be less and many students can still live at home or receive more support from family. Students who attend school locally can stay close to their friends and family. The drawback to staying local can be not gaining/developing as much independence and not making as much of an effort to meet new people and try new things. Compare the resources available at the colleges you are considering. Sometimes larger schools have more resources available.
Paul: It's affordable and that's a huge benefit! The only reason I could move 2000 miles away to live in a new place for school was because of a generous scholarship.
Q: Which is the preferred way of connecting a circuit, series or parallel?Matt: I'm not sure what you mean by circuit, so I'm thinking of solar panels in my answer. Great question by the way. So the preferred method would be determined by the device(s) you are powering. Let's say each panel generates 1.5 volts (same as one AA battery), and let's say each panel can put out 2 amps (that's the actual electricity that runs through the device). If you hook them in parallel you would still only get 1.5 volts, but you would generate 4 amps of electricity. If you hook them in series you would get 2 amps at 3 volts. So if you have a device that needs more voltage (think of something that uses 4 batteries rather than 2) you would want to hook things up in series. If you have something that doesn't need so much voltage and needs more electricity (think of something that drains your batteries very fast) then you want to hook things up in parallel. Thanks for asking!
Q: Is there any advice that helped you guys go through college? Anything you would highly recommend (from classes to friends or any advice)?Natalie: Definitely look at local scholarships, which come out in a packet, at least in Los Alamos. Don’t overlook the small ones either, they helped pay for a new laptop and a desk for me. There are a lot for people going into STEM. And start on them early. The questions are pretty generic, so I recommend looking at past years questions and start this summer. College applications are pretty time consuming but do not burn out before scholarships. LANL typically has an information day for their scholarship, and do not miss it. They give out deadlines, how to apply, and advice. You do not have to be an intern to apply. Girl Scouts offers a lot, including a $20,000 scholarship to be a National Gold Award recipient, and they also have a website full of other scholarships. Hope this helps!
Ashlee: I would recommend trying different types of classes. It is okay to change majors midway through! Exploration is key - it never hurts to try. I tried on-campus research, industry internship, and interning at LANL - all of which had very different experiences. The more you try, the more you will figure out what you like and what you don't like. I also joined different clubs and that was both good experiences and a fun way to make friends. However, don't overwhelm yourself. I did a lot my first couple of years in college (heavy courseload, many clubs) that it was hard to enjoy things or really explore what I liked. I would make sure to save some time for yourself (it will be hard, but you'll find a good balance as you try things).
Tiffany: When I first started I tried to just focus on classes and put a lot of stress on myself; I went to a college out of state, far from home, my parents couldn't afford to help me, and worked hard to get to a university with some sort of scholarship. I didn't want to 'mess' it up. I remember being really upset the first time I got a "B" and thought I'd have to work harder and that can put a lot of stress on oneself. However, even with some B's on my transcripts, I had good relationships with teachers who saw I worked hard, and obtained good recommendations, and those can mean a lot to get summer internships.
I also think extracurricular activities show you're outgoing and motivated, and aren't highlighted enough that they're important on resumes, especially in undergraduate work. They're also a great way to make friends. I joined a climbing club at one point and I made lifelong friends there. It also helped me destress.
Summer internships are a great way to learn what you do and don't like. I did three (very) different ones during undergraduate and they really helped me find my path forward. Apply even if you don't think you'll get it; if you don't try you'll never know. I was always told you couldn't get into Sandia without knowing somebody, but one summer I just happened to have the skills somebody needed, and I got the posting. Most (science) jobs understand that you may not have a lot of 'skills'; hiring undergrads allows us to help you obtain skills, and helps groups give back to the community. This is part of where showing off those hobbies and extracurricular activities on your resume can come in handy
The last piece of advice is to balance your class load. One semester I saw a road to take out a ton of important classes that counted towards my major. I ended up with three (advanced) math classes, two physics classes and one astrophysics class. I don't think I had a single "filler". It was the worst semester of my life and by the end, I was happy to just pass my classes that semester. Try to keep some 'basic' classes. They can be a lot of 'busy' work, but having a change of pace from your more intense major classes is really nice. Even a basic dance or workout class is a great way to get your mind off the harder work and destress.
Isabella:With regards to the first question, some advice that helped me go through college is go to office hours! It might seem kind of scary to go to a professor's office hours, but it is so, so helpful and gives you the chance to ask questions you could not ask during lecture. You can go to TA office hours as well. Additionally, make sure you are involved with your department; that can range from joining clubs to helping out on academic committees, or simply going to department events. This really helped me feel a sense of community and belonging, and it provided me with a lot of opportunities I would otherwise not have been exposed to. Stemming off of that, it's really important to find something non-STEM that you like to do, and force yourself to make time for it! It can be really easy to get overwhelmed by schoolwork and feel like you only have time for school. I promise you will be happier and more productive if you make time for hobbies and socialization. Prioritize your mental health! Hannah: I recommend making time for your hobbies and activities you enjoy. There will be times when school is like a firehose and you may have to focus all your time on it, but it is really helpful to have something non-academic to go back to. For me, that was symphony and taekwondo, which was where I made a lot of friends. It can help to make a nice meal every week or two, taking some time to take care of yourself.
Something that took me a while to really grasp is that professors are there to help you, not just grade you. If you are having issues, go to office hours, send emails, ask for help. In my experience, most professors really want their students to be excited and engaged, so they are more than willing to spend the extra time.
It is really helpful to keep your notes and assignments organized in case you need to refer back to them later. A lot of classes build on each other, and it can save a lot of time to have examples of what you learned a year ago. A few of my classes allowed a note sheet on the final, which was typically a single piece of paper - I ended up using those notesheets over and over again while reviewing for other classes because it was basically like a tiny summary of the class and all the important equations. If you decide to go to grad school, those notes will probably come in handy again.
I wish I had taken a Linear Algebra class earlier than I did (I took it as a senior, but I started needed the material as a junior).
A final note: if you are struggling, remember you are not alone. Classes can be extremely difficult at times, and it is easy to feel isolated. Help your classmates, get their opinions (obviously only on assignments where it is allowed). I found having a study group really helped me make friends, inspired me to do the homework in a timely manner (so I would have a better idea what was going on when we studied), and helped me understand the material at a deeper level.
Cassandra: Take advantage of all the resources available to you in college! Interact with your professors and academic advisor. Get involved in organizations doing work you are passionate about. Use the resources available through the Career Center, Tutoring Center, and Library, and any other student support offices. In addition to getting excellent resources, you will also have the opportunity to network and meet amazing people who can support you throughout your studies and beyond. Consider studying abroad for a semester. In many cases the credits you earn will fulfill degree requirements. I also highly recommend learning a foreign language. Take the time to learn how to navigate your college's academic policies and processes (e.g. registration dates, drop dates, withdraw dates, course repeat policies, payment and refund schedules, etc.). Learn your degree requirements. Do not solely rely on someone else to pick your classes or make sure you are meeting your degree requirements. Appreciate all of your courses, even those that are not directly related to your major. Be strategic and select non-major courses that will complement your major. It is important to be well-rounded and develop transferable skills.Visit your professors during their office hours to discuss the things you are learning in class and ask questions. If you need help in a class, get itimmediately and do not wait. Consider a double major or minor if you are having trouble deciding between two degrees. If you decide early enough, it is usually possible to do both without adding to the amount of time you are in school.Enjoy the journey! It is important to be proactive and involved but also find balance so you can be present in the experience and not be too stressed to enjoy your time in college. Patrick: Be wise when selecting you class load. Signing up for 18+ in a semester is significant.Realize college in like a job. It is up to you to have the discipline to attend class and do the work. It’s unlikely you will have someone reminding you to get up and around for class, do your homework, study. College is a challenge, but don’t get frustrated, stick with it and achieve your goal.Take advantage of Professor office hours; usually posted on, or near their office door (find that location). Don’t be afraid to go to them to ask questions. A semester moves quickly and if you miss something or don’t understand something, it will pass quickly and the confusion can compound.Study groups are good; at the same time be self-reliant on learning the material…they won’t be taking the test for you.Think about residing on campus early in your college career. Getting accustomed to being on your own while still being a student will help by being in a dorm versus an apartment.
Paul: Get involved in activities and groups outside of your classes. Studying and classwork is important, but one of the best parts of college is getting to meet a whole world of new people. Plus it's always nice to have friends and activities that are separate from the classes and subjects you are thinking about all day - variety is the spice of life! Oh and don't be afraid to ask for help: study groups, office hours, and tutoring are not a sign of weakness!
Q: Is there any scholarships for high school students? Do you know reliable websites or places where high schoolers can apply?Ashlee: For high schoolers, there are many scholarships out there. The company that administers the SAT (CollegeBoard) has a scholarship database where you can search through. In addition, a big scholarship is QuestBridge, which is aimed for financially needy students. QuestBridge has a college prep program for high school juniors to help them through the application process. They also have a college scholarship for high school seniors to apply for. CollegeBoard Scholarship Search: https://pages.collegeboard.org/scholarship-opportunities-from-collegeboardQuestBridge: https://www.questbridge.org/Also, applying for financial aid (FAFSA) can lead to scholarships (sometimes called 'grants' - you do not have to pay these back!). However, they might also offer loans. You do not have to accept everything (you could just accept the scholarship/grant). Cassandra: High school seniors who attend high school in northern New Mexico can apply for the LANL Foundation Scholarship.
Paul: I would first ask your high school counselor; they will know the most about local and regional scholarship opportunities that can be hard to find online. But there are also national organizations that might be able to help with scholarships or reduced tuition:American Association of University Women (AAUW): I have heard the local chapters often have scholarship opportunitiesWestern Undergraduate Exchange (WUE): reduced tuition if enrolled at certain universities and degree programs. Check out their College Savings FinderSociety for Advancement of Chicanos/Hispanics and Native Americans in Science (SACNAS): local chapters at Northern NM College and UNM
Q: To Patrick Garcia: did you ever feel overshadowed by your past relatives? If you did what did you do to "surpass" them?Patrick:Keep in mind the time in history were are discussing. My grandfathers were supporting the very early constructing of the laboratory and the United States were in the middle of World War II (WWII) in the 1940s. My family has many, many generations living in the Pojoaque Valley and at the time of WWII in general, a High School graduate was pretty-well educated. My grandfathers were each a heavy machine operator and carpenter. There is nothing wrong with either of those professions and the served a real need at that time. One grandfather had a HS diploma the other didn’t. Now on to my parents who both worked at the Laboratory; they both attended UNM. My mother graduated with an undergraduate degree in HS education and my father was drafted to the US Army and did not complete a degree. At the Laboratory my mother’s role was a Human Resources generalist and my father was responsible for a warehouse providing materials and components for experiments and testing to support the mission of the Laboratory during the Cold War. Based on their experiences and interactions at the Laboratory, they both instilled the importance of getting a college education. My older siblings and I all received college degrees (brother – Civil Engineering; sister – Finance) with mine being a BS-level degree in Mechanical Engineering. That degree enabled me to get the job I have and not be in a shadow of either my parents or grandfathers, simply because our roles and responsibilities were/are very different. At the Laboratory, I’ve advanced in position simply by doing the job I had at the time to the best of my ability and looked for opportunities to advance when I was ready…having the experience and technical background and then applying! Make your own path and cast your own shadow. Those actions are within your control. What others have done or have received recognition can be a metric, but is not the be-all, end-all. Success of an individual’s career is measured by the individual…are you satisfied you did all you could do when you look in the mirror. I encourage you to work hard now in school to learn and become technically knowledgeable, so you don’t have to work hard physically in the future (hope that makes sense). I’m happy to continue discussion or clarify my response if needed.

Thursday June 17

Q: Can you force the genes to turn on/off purposely to produce certain traits?A: Yes, you can use the new technique (invented by women, including my colleague Jennifer Doudna) Crispr/Cas-9 to add or remove genes to alter certain traits. [Karissa]
Q: How do you transfer the knowledge on your computer onto a robot such as the Roomba?A: It depends on the robot! Often the robot has specialized computer hardware so that it can run more efficiently. We usually train a machine learning model on our traditional computers and then transfer it to the specialized hardware on the robot.[Katie]
Q: Can you store the power that you get from the solar panels?
Q: Is nuclear fission the best source of renewable energy?A: I won't want to say it is the "best", as we are still in the research development stage. We don't even know it will work or not for real, therefore, it is too soon to claim it is the best renewable energy. [Ning]
Q: Ning said she studied how Aluminium could be linked to Alzheimers, what results did she find?Although elevated level of aluminum was detected in some patients who were diagnosed with Alzeirmer's diseases, it is definitely not the only factor that caused this brain dysfunction problem. In addition, how does the aluminum get into the brain is not very clear. Regardless, some regions of the world has considered to lower the aluminum level in their drinking water standard. I have attached a link of a summary paper published a couple of years after the study we conducted at the medical School at the University of Kentucky. https://onlinelibrary.wiley.com/doi/pdf/10.1002/env.3170060303 [Ning]
Q: Is there another isotope that can be used to power the rover?A: Yes, both Americium-241 and Polonium-210 can be and have been used in the deep space explore mission. But plutonium-238 is the only one used to power the Mars rover. [Ning]
Q: How do you detect solar storms and why can you only detect it when it is already so close to Earth?
Q: What kind of tests do you do to test for impurities in the plutonium?
Q: What is the difference between UNM and NMSU?
Q: Is it possible to do two years at CNM then go to NM Tech?
Q: Can you get scholarships for being an intern?
Q: Does LANL offer internships for anything medical related?
Q: What lead Megan to choose Perdue besides the fact that it was far from California (in terms of academics)?
Q: What about international colleges? Do you recommend them?

ASK ANNA Questions from 2019 Summer Camp

-Question: Although Global warming is happening, why are some areas in the world getting colder?

https://www.forbes.com/sites/startswithabang/2019/01/30/this-is-why-global-warming-is-responsible-for-freezing-temperatures-across-the-usa/#34272b0ed8cf

“When the vortex at the north pole becomes extremely weak, the high pressure zones found in the middle latitudes of Earth (where the westerlies are) can push towards the poles, displacing the cold air. This causes the polar vortex to move farther south. In addition, the jet stream buckles, and deviates towards more populous, southern latitudes. As the cold, dry air from the poles comes in contact with the warm, moist air of the mid-latitudes, you get a dramatic weather change that we conventionally refer to as a cold snap.

https://www.nationalgeographic.com/environment/2019/01/climate-change-colder-winters-global-warming-polar-vortex/

“First, it's important to understand the difference between climate and weather. Climate is defined as the average weather patterns in a region over a long period of time.”

“Scientists believe Earth will experience more extreme, disastrous weather as the effects of climate change play out.”

“As more Arctic air flows into southern regions, North America can expect to see harsher winters. That was the conclusion of a study published in 2017 in the journal Nature Geoscience. It found a link between warmer Arctic temperatures and colder North American winters. A separate study published in March of last year in the journal Nature Communications found the same link but predicted the northeastern portion of the U.S. would be particularly hard hit.”

https://www.nature.com/articles/ngeo2986#f4

“Warming temperatures in the Northern Hemisphere have enhanced terrestrial productivity. Despite the warming trend, North America has experienced more frequent and more intense cold weather events during winters and springs. These events have been linked to anomalous Arctic warming since 1990, and may affect terrestrial processes. Here we analyse multiple observation data sets and numerical model simulations to evaluate links between Arctic temperatures and primary productivity in North America. We find that positive springtime temperature anomalies in the Arctic have led to negative anomalies in gross primary productivity over most of North America during the last three decades, which amount to a net productivity decline of 0.31 PgC yr−1 across the continent. This decline is mainly explained by two factors: severe cold conditions in northern North America and lower precipitation in the South Central United States. In addition, United States crop-yield data reveal that during years experiencing anomalous warming in the Arctic, yields declined by approximately 1 to 4% on average, with individual states experiencing declines of up to 20%. We conclude that the strengthening of Arctic warming anomalies in the past decades has remotely reduced productivity over North America”

https://science.sciencemag.org/content/343/6172/729

“a rise in global mean temperature will almost certainly lead to an increase in the incidence of record high temperatures. Global warming also leads to increases in atmospheric water vapor, which increases the likelihood of heavier rainfall events that may cause flooding. Rising temperatures over land lead to increased evaporation, which renders crops more susceptible to drought. As the atmosphere and oceans warm, sea water expands and glaciers and ice sheets melt. In response, global sea-level rises, increasing the threat of coastal inundation during storms.”

https://agupubs.onlinelibrary.wiley.com/doi/full/10.1029/2012GL051000

“two hypothesized mechanisms by which Arctic amplification – enhanced Arctic warming relative to that in mid‐latitudes – may cause more persistent weather patterns in mid‐latitudes that can lead to extreme weather. One effect is a reduced poleward gradient in 1000‐500 hPa thicknesses, which weakens the zonal upper‐level flow. According to Rossby wave theory, a weaker flow slows the eastward wave progression and tends to follow a higher amplitude trajectory, resulting in slower moving circulation systems. More prolonged weather conditions enhance the probability for extreme weather due to drought, flooding, cold spells, and heat waves. The second effect is a northward elongation of ridge peaks in 500 hPa waves, which amplifies the flow trajectory and further exacerbates the increased probability of slow‐moving weather patterns. While Arctic amplification during autumn and winter is largely driven by sea‐ice loss and the subsequent transfer of additional energy from the ocean into the high‐latitude atmosphere, the increasing tendency for high‐amplitude patterns in summer is consistent with enhanced warming over high‐latitude land caused by earlier snow melt and drying of the soil.”

“We find that a warmer Arctic atmosphere contributes to dilated geopotential heights locally accompanied by lower heights across mid-latitudes and an equatorward-shifted jet stream. This allows Arctic airmasses to expand farther south while increasing the likelihood of heavy snowfalls. We find a distinction between early winter, when Arctic warming tends to affect only the lower troposphere, and mid-winter to late-winter when polar cap geopotential height anomalies is evident throughout the troposphere and lower stratosphere. When the entire Arctic atmospheric column is affected, the probability of severe winter weather in mid-latitudes increases, as observed during the era of AA in late winter. Colder Arctic conditions elicit the opposite response. These findings suggest that the continuation of rapid Arctic warming and melting contribute to more frequent episodes of severe winter across the Northern Hemisphere mid-latitude continents.”

Dan Reisenfeld answered some of your questions:

1) Q: do tsunamis and earthquakes happen in space?” I am taking it to mean do they happen on other planets or moons. Of the planets and moons in our solar system, Earth is the only one with tsunamis, at least in recent geologic times. Other planets have been measured to have small seismic activity, in particular Mars definitely has “Marsquakes”, although much smaller than on Earth. In fact there is a mission on Mars right now called InSight, whose function is to look for Marsquakes. A moon of Jupiter, Io, has active volcanoes (the only other place in the solar system to have them besides Earth), and so likely there are quakes on Io, too. Venus could also have quakes, but we’ve never had instruments on Venus to measure them. It is about the same size as the Earth and could very well have platectonics, like on Earth, which would give rise to quakes.

2)Q: What gives the planets their colors? A: The reason depends on the planet.

  • Mercury and the Earth’s Moon (I know, not a planet, but kinda prominent in our sky) are grayish because they have no atmosphere and have surfaces covered with grayish rock, sand and dust made up mostly of silica and calcium which are whitish/grey in color.

  • Venus is yellow because its atmosphere has lots of sulfur in it (which is yellow).

  • The Earth is blue-green, because well, oceans and plants.

  • Mars is “red” (really orange) because the rocks on its surface have a lot of iron oxide (literally rust) in them. If you look at the Sahara desert from space, it too looks reddish because of all the iron in the Earth’s crust. So if the Earth were bare of water and plants, it would look a lot like Mars.

  • The outer planets (Jupiter, Saturn, Uranus, Neptune) are all gas giants with atmospheres made up mostly of hydrogen. The colors, though, come from the trace particles in their atmospheres. There are lots of ammonia, phosphorous and methane crystals in their atmosphere that reflect different colors depending on their concentrations and temperatures. Since the outer planets are at different distances from the Sun, the colors are different for the different planets because their temperatures are so different.

  • How do planets get those colors? http://scienceline.ucsb.edu/getkey.php?key=1088

Here are a couple of videos that review the topics we worked on today:

Electricty and Magnetism

https://www.youtube.com/watch?v=hFAOXdXZ5TM

https://www.youtube.com/watch?v=ru032Mfsfig

How does electricity work? http://scienceline.ucsb.edu/getkey.php?key=255

How did electricity start: http://scienceline.ucsb.edu/getkey.php?key=400

How does the atomic structure influence the PH?

https://study.com/academy/lesson/how-acid-base-structure-affect-ph-pk-values.html

“Acids exist in an equilibrium with their conjugate base. The strength of the acid (pKa) depends on the stability of the base. When the proton leaves the acid, it leaves behind its electrons. Those are super negative and there is a big negative charge on the conjugate base.

You know that if there is a concentrated negative charge, the base is not very stable. However, if the charge can be spread out, then the base is more stable, which means we have a stronger acid.“

“the stronger acid has a more stable base. One thing that influences base stability is the size of the ion. A larger ion can accommodate a negative charge better. Imagine that the charge has more 'room' to spread out.

Luckily, we can use periodic trends to predict the size of the ion. This means we can predict the more stable base and therefore, the stronger acid. Ion size increases as we go from top to bottom of a column on the periodic table. (Ion size also increases from right to left, but that does not influence base stability due to electronegativity, which we'll talk about in a moment). “

LANL Visit Questions:

  • Vivien, Scott

One of the girls asked the following question and I was wondering if you would like to address it: "understanding how a generator that big powers a magnet, like what's the different things it does in order to power"

The generator is very similar to the little generators we played with during the summer camp. It has three magnets that rotate inside a coil of wire. The moving magnets creates electricity in a coil of wire, which in turn powers the magnet.

We need the generator because the power grid can’t deliver as much energy as we need quickly enough. So we gather energy off the power grid slowly for an hour and store it, and then deliver it to the magnet in a second.

We gather that energy by using a motor to slowly spin up the generator. Energy is stored in the rotation of its massive shaft. Then we turn around and deliver that energy in the form of electricity to the magnets in just a second.

Our magnets use only a little bit of energy (the biggest magnets use as much as my house does in a day for each 1 second pulse from 0 to 100 Tesla and back to 0). But the power the magnets use (how fast energy is delivered) is as much as an entire city.